You are on page 1of 70

Matematicas Discretas I

por Mara Luisa Perez Segu


Introduccion
Se presenta aqu el material correspondiente a un primer curso en Matematicas Discretas,
el cual se imparte en la Facultad de Ciencias Fsico-Matematicas de la Universidad Michoa-
cana. El material del libro constituyo el 100 % del curso impartido el semestre de agosto de
2015 a enero de 2016 en la Facultad. Parte fundamental del aprendizaje de las Matematicas
Discretas esta basado en la resolucion de problemas. Por esta razon, se intercalan numerosos
ejemplos inmediatamente despues de que se ha introducido un concepto nuevo, de manera
que puedan practicarse las tecnicas de resolucion. Se proponen tambien diversos ejercicios;
algunos de ellos son rutinarios, mientras que la solucion de otros requiere de un mayor es-
fuerzo, imaginacion y dedicacion. Buena parte del material fue una adecuacion de los dos
libros de Combinatoria de la autora, los cuales se utilizan intensamente en etrenamientos
para la Olimpiada de Matematicas en todo el pas.
La seccion 1 proporciona el material basico del conteo. Se tratan aqu las formulas de
permutaciones y combinaciones. Se explican tambien las tecnicas de multiplicacion y las de
separacion en casos.
La seccion 2 estudia el Teorema del Binomio.
En la seccion 3 se prueban varias formulas importantes en la Combinatoria. El metodo
usado es el de comparaciones, que consiste en contar de dos (o mas) formas distintas lo
mismo.
En la seccion 4 se explica el metodo de demostracion por induccion. Este es un metodo
basico en un gran numero de demostraciones de matematicas discretas y tiene aplicaciones
y metodos de aplicacion muy diversos. Se dan ejemplos de todos estos metodos y se explican
cuidadosamente los errores que comete en general un principiante.
En la seccion 5 se tratan algunos ejemplos en los que la idea es contar caminos con ciertas
condiciones. El tema se presta para practicar tecnicas de conteo.
La seccion 6 provee otro metodo de conteo importante, consistente en sumar y despues
compensar restando cuando hubieran aparecido repeticiones.
La seccion 7 es una muy breve introduccion a la probabilidad matematica. Simplemente
se establecen las bases y se estudian algunos ejemplos.
En la seccion 8 se estudia un principio muy basico y, a primera vista, elemental, que es el
Principio de las Casillas, y que simplemente establece que si se quieren colocar en cajas un
numero de objetos mayor que el de cajas, entonces al menos una caja debera tener mas de un
objeto. Las aplicaciones de este principio son muy diversas y algunas de ellas son bastante
complicadas. La resolucion de problemas de esta ndole abre un panorama interesante en el
razonamiento matematico.
La seccion 9 estudia el tema de coloracion, el cual simplemente consiste en distinguir
unos objetos de otros; dicha clasificacion ofrece nuevas tecnicas de conteo.

i
Finalmente, en la seccion 10 se da una introduccion al importante tema de las graficas
(o grafos). Este tema es esencial en el estudio de la computacion. Por esta razon, se trata
de abarcar los principales conceptos y resultados mas basicos, incluyendo paseos eulerianos
y ciclos hamiltonianos, planaridad y la formula de Euler, y coloracion de mapas.

Mara Luisa Perez Segu


Fac. Cs. Fsico-Matematicas
Universidad Michoacana de San Nicolas de Hidalgo
diciembre, 2015.

ii
Indice
Introduccion I

1. Conteo 1

2. Teorema del binomio 12

3. Comparaciones 14

4. Induccion Matematica 16

5. Caminos 28

6. Inclusion y Exclusion 31

7. Probabilidad Combinatoria 34

8. Principio de las Casillas 41

9. Coloracion 46

10.Graficas 50
10.1. Paseos eulerianos y ciclos hamiltonianos . . . . . . . . . . . . . . . . . . . . 55
10.2. Graficas planas . . . . . . . . . . . . . . . . . . . . . . . . . . . . . . . . . . 59

iii
1. Conteo

Uno de los conceptos matematicos abstractos mas primitivos que conocemos es el de


numero y, dentro de los numeros, el de los numeros naturales enteros positivos: 1, 2,
3, 4, etc. Con ellos representamos las cantidades de objetos que se nos presentan en la vida
cotidiana. En esta seccion desarrollaremos algunas tecnicas que permiten determinar con
facilidad cantidades. Comencemos por ilustrar la necesidad de aprender estas tecnicas de
conteo con unos ejemplos: Si se nos ensena un punado de canicas y se nos pregunta cuantas
son, un vistazo nos bastara para contarlas y dar la respuesta; sin embargo si se nos pregunta
cuantas patas tienen 100 perros, en lugar de buscar los 100 animales y contarles las patas,
haremos una abstraccion, y la operacion: 4 100 = 400 nos dira la respuesta; utilizamos aqu
una tecnica muy simple de multiplicacion. Desde luego hay preguntas que necesitan tecnicas
mas elaboradas. Estudiaremos estas tecnicas mediante ejemplos que iremos complicando
gradualmente.

Analicemos primero con cuidado un ejemplo que a primera vista es trivial pero que nos
ensena la clave basica del conteo.

1.1 Ejemplo. Cuantos numeros enteros de tres o menos cifras hay?


Solucion. La respuesta a esta pregunta es facil: Hay 1000 pues son todos los numeros
enteros del 0 al 999. Esta solucion no nos ensena gran cosa. Retomemos ahora el problema
buscando una solucion constructiva; esto es, para cualquier n = 1, 2, 3, . . ., la cantidad de
numeros de hasta n + 1 cifras se puede obtener de la cantidad de numeros de hasta n cifras:
simplemente se multiplica por 10. Vamos a describir con detalle este procedimiento:
Numeros de a lo mas una cifra hay 10, a saber, 0, 1, 2, 3, 4, 5, 6, 7, 8 y 9. Para contar los
de hasta dos cifras (del 0 al 99) no necesitamos escribirlos todos; basta con observar que la
primera cifra puede ser cualquiera de los 10 dgitos 0, 1, 2, 3, 4, 5, 6, 7, 8, 9, y por cada uno de
estos hay 10 terminaciones distintas; por ejemplo, los numeros de dos cifras que empiezan con
4 son: 40, 41, 42, 43, 44, 45, 46, 47, 48 y 49, diez en total; lo mismo para cada una de las otras
decenas. As la cantidad de enteros entre 0 y 99 es 1010 = 100. El siguiente paso es analogo:
Para contar los numeros de hasta tres cifras hay que agregar un dgito (posiblemente 0) a
cada uno de los 100 numeros de 2 o menos cifras; como hay diez posibilidades la respuesta
sera 10 100 = 1000.

Este procedimiento de construir sobre lo ya construido que hemos utilizado se llama


procedimiento inductivo . Muchas demostraciones de propiedades y formulas de numeros
naturales se basan en el. Mas adelante se estudiara esto con detalle. El principio combinatorio
que manejamos en el ejemplo anterior (y que manejaremos en los siguientes) es:

1.2. Principio Fundamental de Conteo. Si una cierta tarea puede realizarse de m


maneras diferentes y, para cada una de esas formas, una segunda tarea puede realizarse de
n maneras distintas, entonces las dos tareas juntas pueden realizarse (en ese orden) de mn

1
formas diferentes.

1.3 Ejemplo. Cuantas palabras de tres letras se pueden formar si se dispone de un


alfabeto con dos letras: a y b. (Nota: Son permisibles palabras como bba.)
Solucion. Procederemos como en el ejemplo anterior. En este caso conviene ilustrarlo
haciendo un diagrama arbol:

Resolvamos ahora el ejemplo utilizando nuestro Principio Fundamental de Conteo. Con-


sideremos tres casillas: , la primera para la letra inicial, la segunda para la letra central
y la tercera para la letra final. En cada casilla hay dos elecciones posibles: la letra a o la
letra b. La respuesta es entonces 2 2 2 = 8. El procedimiento inductivo es como sigue:
En la primera casilla hay 2 posibilidades para elegir la letra. Una vez formada una palabra
de una letra: a o b, para agrandarla a una palabra de dos letras hay dos posibilidades, as
que palabras de dos letras hay 2 2 = 4. Para completar cada una de estas a una palabra
de tres letras hay dos posibilidades; entonces hay 4 2 = 8 palabras de tres letras.

1.4 Ejemplo. Cuantas placas distintas hay con dos letras a la izquierda y tres numeros
a la derecha? (Nota: Consideraremos el alfabeto de 27 letras castellanas.
Solucion. Seguimos el procedimiento de las casillas del ejemplo anterior:

27 27 10 10 10 = 729 000.
| {z } | {z }
lugares lugares
para letras para numeros

1.5 Ejemplo. Cuantas banderas bicolores se pueden formar si se dispone de 4 lienzos


de tela de colores distintos y un asta? (Nota: Banderas como rojo-rojo no son permisibles;
por otro lado, es importante el color que queda junto al asta, de esta manera banderas como
rojo-azul y azul-rojo se consideran distintas.)

2
Solucion. En este caso consideramos dos casillas. La de la izquierda, digamos, representa
el lienzo junto al asta, el cual tiene 4 elecciones posibles. Una vez elegido este, el color para
la derecha se puede escoger de 3 formas (pues no se permite la repeticion de colores). As
hay 4 3 = 12 formas distintas de formar las banderas.

1.6 Ejercicio. Escribir todas las banderas que pueden formarse segun el ejemplo an-
terior si los colores son rojo (R), azul (A), verde (V ) y blanco (B).

1.7 Ejemplo. Misma pregunta que en el ejemplo anterior pero ahora suponiendo que
no hay asta. (En este caso no habra distincion entre las banderas rojo-azul y azul-rojo.)
Solucion. Para resolver este ejemplo analicemos la respuesta del ejemplo anterior. En
aquel, en la coleccion total de las 12 banderas posibles podemos aparear cada bandera con
su opuesta; por ejemplo la bandera azul-verde la apareamos con la bandera verde-azul. Cada
una de las del ejemplo anterior se esta contando dos veces y, por tanto, la respuesta es
12
2
= 6.

1.8 Ejercicio. En el resultado del ejercicio 1.6 aparear cada una de las banderas con
su opuesta. Dar una lista de 6 banderas que ilustre la respuesta del ejemplo 1.7.

1.9 Ejemplo. De cuantas formas se pueden sentar 5 personas en 5 sillas numeradas


del 1 al 5?
Solucion. En el asiento #1 se puede sentar cualquiera de las 5 personas; para cada eleccion
de la primera persona, la segunda puede ser cualquiera de las 4 restantes; as en las dos
primeras sillas el numero de elecciones posibles es 5 4 = 20. Continuamos de manera
analoga. Para simplificar dibujemos 5 casillas simbolizando los 5 asientos. Sobre cada casilla
escribamos el numero respectivo de posibilidades y multipliquemos:
5 4 3 2 1 = 120.

Si n es un numero natural, el producto de todos los numeros naturales del 1 al n aparece


muy frecuentemente en problemas de combinatoria; se llama n factorial o factorial de n
y se denota por n!. (As la respuesta del ejemplo 1.9 es 5! = 120.)

Alejandose de la interpretacion de n! como el producto de los naturales de 1 a n, se define


0! = 1;
esto permite incluir el caso n = 0 en algunas formulas en las que interviene n!. Entonces
0! = 1
1! = 1
2! = 12=2
3! = 123=6
4! = 1 2 3 4 = 24.

3
Es facil darse cuenta que el numero 5 del ejemplo 1.9 y el que sean personas y asientos en
lugar de cualquier otra cosa no es relevante; podemos generalizarlo como sigue:

El numero Pn de distintas formas en que se pueden ordenar n objetos es n!. Cada una de
las listas ordenadas que se forman con los n objetos se llama permutacion (de los objetos).
Tenemos entonces que el numero de permutaciones de n objetos es Pn = n!.

1.10 Ejemplo. De un grupo de 5 estudiantes quiere elegirse una comision de 3 para


que cada uno visite un museo de una lista de 3 museos. Cuantas comisiones distintas se
pueden formar?
Solucion. Utilizando el esquema de casillas (cada una representando un museo) como
arriba, tenemos que el resultado es

5 4 3 = 60.

1.11 Ejemplo. De un grupo de 5 estudiantes quiere elegirse una comision de 3 para


que juntos visiten un museo (el mismo todos). Cuantas comisiones diferentes se pueden
formar?
Solucion. Hay que observar que la diferencia entre este ejemplo y el anterior es que no
importa el orden en la eleccion. En el ejemplo anterior haba disticion entre las casillas pues
cada una representaba un museo en particular distinto a los otros; en este no hay distincion
entre las casillas pues, por ejemplo, una comision en que se haya elegido la sucesion de
alumnos Ana-Beto-Carlos se considerara igual a la sucesion Beto-Carlos-Ana y tambien
igual a la sucesion Ana-Carlos-Beto. Nuestro interes es entonces determinar en la cantidad
5 4 3, en cuantas sucesiones aparece el mismo conjunto de alumnos. Para responder
esto conviene plantear esta parte del ejemplo al reves: Consideremos un conjunto fijo de 3
personas, por ejemplo el formado por Ana (A), Beto (B) y Carlos (C) y contemos de cuantas
formas se pueden ordenar estos 3. Observemos que el numero de formas es precisamente el
numero de permutaciones de las 3 personas, o sea, P3 = 3! = 6. Entonces cada grupo de 3
personas se esta contando 6 veces en el producto 5 4 3, as que la respuesta al ejemplo
sera
543
= 10.
3!

1.12 Ejercicio. En los ejemplos 1.10 y 1.11 supongamos que el grupo de los 5 alumnos
esta formado por Ana (A), Beto (B), Carlos (C), Daniel (D) y Elena (E). Hacer la lista de
los 60 arreglos de estos alumnos en los que se elige 3 para visitar museos distintos, agrupando
en esa lista las colecciones que resultan iguales si todos van a un mismo museo.

En el ejemplo anterior aprendimos el siguiente principio:

1.13. El numero de colecciones (en las que el orden no importa) con r elementos que

4
se pueden seleccionar dentro de un conjunto de n elementos (n r 1) es
n (n 1) (n (r 1))
.
r!

Este numero recibe el nombre de combinaciones de n en r y se denota por nr . Dicho




de otra manera, el numero de subconjuntos de r elementos que tiene un conjunto con n
elementos es nr . (En el ejemplo 1.11, n = 5 y r = 3 y la respuesta es 53 .) Notese que la
formula 1.13 no tiene sentido para n = 0; sin embargo s tiene sentido hablar del numero
de subconjuntos con 0 elementos dentro de un conjunto con n elementos; sabemos que este
numero es 1 pues solo hay un conjunto sin elementos que es el llamado conjunto vaco.
Definimos entonces  
n
= 1.
0

1.14 Ejercicio. Sea X = {a, b, c, d, e}. Escribir todos los subconjuntos de X con
(a) 0 elementos,
(b) 1 elemento,
(c) 2 elementos,
(d) 3 elementos,
(e) 4 elementos y
(f) 5 elementos.
Verificar que en cada caso el numero de subconjuntos obtenido sea 5r y que el numero


total de subconjuntos sea 25 = 32.

1.15 Ejercicio. Basandose en la interpretacion de nr como el numero de subconjuntos




de r elementos dentro de un conjunto con n elementos, explicar por que


   
n n
= .
r nr
1.16 Ejercicio. Calcular 72 , 75 , 55 y 94 .
   

Con la intencion de simplificar la formula 1.13 sobre las combinaciones de n en r, ob-


servemos que, para 1 r n 1, el numerador se puede completar a n! multiplicando
por (n r)!; si lo completamos deberemos compensar dividiendo tambien por (n r)!.
Tendremos entonces que para r = 1, 2, . . . , n 1,
1.17.  
n n!
= .
r r!(n r)!

Recordemos que se ha definido 0! = 1 y n0 = 1; notemos entonces que si sustituimos




r = 0 (y, posiblemente tambien n = 0) en el lado derecho de la formula 1.17 obtendremos

5
n! n!
0!n!
= 1. De la misma manera, al sustituir r = n obtendremos n!0!
= 1. As, tambien en estos
casos extremos vale la formula 1.17.

1.18 Ejercicio. Volver a hacer los ejercicios 1.15 y 1.16 utilizando la formula 1.17.

1.19 Ejemplo. De un grupo de 10 ninos y 15 ninas se quiere formar una coleccion de


5 jovenes que tenga exactamente 2 ninas. Cuantas colecciones  distintas se pueden formar?
15 1514
Solucion. La eleccion de las 2 ninas se puede hacer de 2 = 2! = 105 formas. Como
deben ser 5 en total y debe haber 2 ninas exactamente, entonces los ninos seran 3; estos se
pueden escoger de 10 3
= 1098
3!
= 120 formas. Por tanto el resultado es 105120 = 12 600.

Como hemos visto, al determinar cantidades buscamos simplificar nuestras cuentas uti-
lizando homogeneidades en el problema. Con este proposito, en algunas ocasiones es con-
veniente dividir en casos de manera que en cada uno de ellos haya homogeneidad, y despues
sumar las respuestas. Un ejemplo muy simple de esto sera el siguiente: Si tenemos 4 paquetes
de 100 hojas de papel y otros 3 paquetes de 200 hojas cada uno, entonces el numero total
de hojas que tenemos es
4 100 + 3 200 = 1000.

Comparemos el siguiente ejemplo con el anterior, tomando en cuenta la busqueda de


homogeneidades, como acabamos de decir.

1.20 Ejemplo. De un grupo de 10 ninos y 15 ninas se quiere formar una coleccion de


5 jovenes que tenga a lo mas 2 ninas. Cuantas colecciones distintas se pueden formar?
Solucion. Vamos a resolver este ejemplo como el anterior pero separando por casos y
despues sumando las respuestas de cada uno de los casos.
Caso 1: Que la coleccion tenga 2 ninas exactamente: 15
 10
2
= 12 600.
 3
15 10
Caso 2: Que la coleccion tenga exactamente 1 nina: 1 4 = 3 150.
Caso 3: Que la coleccion no tenga ninas: 15
 10
0 5
= 252.
La respuesta al ejemplo es 12 600 + 3 150 + 252 = 16 002.

1.21 Ejemplo. Un grupo de 15 personas quiere dividirse en 3 equipos de 5 personas


cada uno. Cada uno tendra una labor especfica distinta a las demas. De cuantas formas
distintas es posible hacer la distribucion?
Solucion. Escojamos uno por uno los equipos. La eleccion del primer equipo puede hacerse
15

de 5 = 3 003 formas; para elegir el segundo equipo ya solo habra 10 personas de donde
escoger, por tanto este se podra elegir de 10

5
= 252 formas. El tercer equipo quedara formado
automaticamente con la eleccion de los otros dos. Entonces el numero de formas de hacer la
eleccion sucesiva es 3 003 252 1 = 756 756.

1.22 Ejemplo. Un grupo de 15 personas quiere dividirse en 3 equipos de 5 personas

6
cada uno. Todos los equipos tendran la misma labor. De cuantas formas es posible hacer la
distribucion?
Solucion. En este caso no hay distincion entre los equipos as que hay que dividir el
resultado del ejemplo anterior entre 3!, que es el numero de permutaciones de los equipos.
La respuesta es entonces 126 126.

1.23 Ejemplo. En una bolsa hay 3 pelotas rojas y 2 azules. Se quiere formar una fila
con todas ellas. De cuantas maneras distintas puede quedar la fila?
Solucion. Primera forma. Consideremos todas las permutaciones de las 5 pelotas y con-
temos cuantas de esas permutaciones son indistinguibles entre s. Las permutaciones de las
5 pelotas sabemos que son 5! = 120. En cualquiera de las permutaciones fijemonos en la
ubicacion de las pelotas rojas; por ejemplo roja roja roja. estas pueden revolverse
entre s (3! veces) formando colecciones indistinguibles, y lo mismo ocurre con las del otro
color. Vamos a explicar lo anterior con mas detalle: Denotemos las pelotas rojas por R1 , R2
y R3 , y las azules por A1 y A2 . Entonces las siguientes listas (en las que se han permutado
las rojas pero se han dejado fijas las azules) representan la misma coleccion:

A1 R1 A2 R2 R3
A1 R1 A2 R3 R2

A1 R2 A2 R1 R3
A1 R2 A2 R3 R1 .


A1 R3 A2 R1 R2
A1 R3 A2 R2 R1

Estas 3! listas deben considerarse como una sola. Ademas, en cada una de ellas tambien se
pueden revolver las azules entre s (2! permutaciones). Entonces al considerar las permuta-
ciones de las 5 pelotas, cada arreglo se esta contando 3! 2! = 12 veces en lugar de 1. La
5!
respuesta al ejemplo es pues 3!2! = 10.
Segunda forma. Primero podemos contar las posibilidades para colocar las pelotas rojas
en los 5 lugares disponibles; esto nos dara la eleccion de 3 lugares, que puede hacerse de
5
3
= 10 maneras. Para colocar las 2 azules ya solo sobran 2 lugares as que esto se puede
2

hacer de 2 = 1 forma. El resultado es 10 1 = 10.

1.24 Ejercicio. Escrbanse las 10 filas distintas que se pueden formar con las pelotas
en el ejemplo 1.23.

1.25 Ejemplo. En una bolsa hay 3 pelotas rojas y 2 azules. Cuantas filas distintas
de 3 pelotas se pueden formar?
Solucion. Como son 5 pelotas en total pero solo se van a considerar filas de 3, hay que dejar
dos pelotas sin colocar. Consideraremos los distintos casos por separado y despues sumaremos
3!
las respuestas parciales. Si las dos pelotas que quedan fuera son rojas, hay 1!2! = 3 arreglos
3! 3!
con las restantes. Analogamente hay 3! = 1 fila que deja las 2 pelotas azules fuera, y 2!1! =3

7
filas que dejan una azul y una roja fuera. La respuesta al ejemplo es 3 + 1 + 3 = 7.

1.26 Ejercicio. Escribir los 7 arreglos de pelotas del ejemplo 1.25 .

En algunas ocasiones, para poder hacer bien las cuentas, nuestra busqueda de homoge-
neidad nos lleva a que es mas facil contar lo opuesto de lo que queremos y despues restar de
un total. Ilustramos esto con el siguiente ejemplo.

1.27 Ejemplo. De cuantas maneras pueden ordenarse en un estante 3 cuadernos


rojos, 4 azules y 2 verdes, si los verdes no deben quedar juntos?
Solucion. Conviene contar primero todas las ordenaciones posibles y despues restar aquellas
en las que los verdes quedan juntos. El numero total de filas (incluyendo aquellas en que los
9!
verdes quedan juntos es 3!4!2! = 1260. Para contar las que tienen juntos los cuadernos verdes
pensemos estos como pegados formando un solo cuaderno; ahora determinemos el numero de
8!
arreglos con 3 cuadernos rojos, 4 azules y 1 verde; este es 3!4! = 280. La respuesta al ejemplo
es 1260 280 = 980.

1.28. Los ejemplos siguientes se refieren a la baraja usual de pokar: Cada carta tiene
un smbolo llamado numero que puede ser cualquiera de los 13 smbolos siguientes: A, 2,
3, 4, 5, 6, 7, 8, 9, 10, J, Q o K, y otro smbolo llamado palo que puede ser cualquiera de
los 4 siguientes: (espada), (corazon), (diamante) o (trebol). Todos los palos
se combinan con todos los numeros para formar la baraja completa con 13 4 = 52 cartas
como se ilustra a continuacion:

A 2 3 4 5 6 7 8 9 10 J Q K
A 2 3 4 5 6 7 8 9 10 J Q K
A 2 3 4 5 6 7 8 9 10 J Q K
A 2 3 4 5 6 7 8 9 10 J Q K
Se llama mano de pokar cualquier coleccion de 5 cartas de la baraja. La siguiente nomen-
clatura es usual:
par: dos cartas del mismo numero.
tercia: tres cartas del mismo numero.
pokar: cuatro cartas del mismo numero.
full: una tercia y un par.
flor: cinco cartas del mismo palo.
corrida: cinco cartas con numeracion consecutiva (segun el orden en que se escribieron
arriba, pero permitiendo A tambien como numero final, en seguida de K).
Observemos que el numero total de manos de pokar es 52

5
= 2 598 960.

1.29 Ejemplo. Cuantas manos de pokar tienen tercia exactamente (es decir, que no
sea full ni pokar).

8
Solucion. Primera forma. Ponemos 5 casillas: las tres primeras para la tercia y las otras
dos para las otras cartas. La primera carta se puede escoger arbitrariamente; la segunda solo
tiene 3 posibilidades pues debe tener el mismo numero que la primera; la tercera ya solo
puede ser elegida de 2 maneras distintas; como no importa el orden de estas 3 cartas, este
numero debera dividirse entre 3!. La cuarta carta se debe escoger dentro de las 48 que son de
numero distinto al de la tercia. Para la quinta carta ya solo sobran 44 cartas pues el numero
debe ser tambien distinto. La cuarta y quinta pueden haberse escogido en cualquier orden
por lo que se debera dividir entre 2!.
52 3 2 48 44
= 54 912.
| {z3! } 2! }
| {z
tercia cartas distintas

Segunda forma. Tambien formamos primero la tercia pero eligiendo antes el numero que le
correspondera: Tenemos 13 numeros para escoger y, una vez escogido el numero, las 3 cartas
que forman la tercia deben escogerse dentro de 4 posibles; entonces el numero de tercias
es 13 43 . Para escoger las otras dos cartas utilizando este mismo metodo razonamos como


sigue: Hay que escoger 2 numeros (pues queremos que las otras 2 cartas sean de numeros
distintos) dentro de los 12 que sobran; esta eleccion se puede hacer entonces de 12
2
formas.
En cada uno de estos numeros que se hayan elegido hay que escoger 1 carta, cosa que puede
hacerse de 41 formas. El resultado escrito en esta forma es
    2
4 12 4
13 ,
3 2 1

que, desde luego, tambien es igual a 54 912.

1.30 Ejemplo. Cuantas manos de pokar tienen dos pares (distintos) exactamente?
Solucion. Procedemos como en el ejemplo 1.29.

Primera forma.
1er par 2o par
z }| { z }| {
52 3 48 3
2! 2! 44 = 123 552.
2!
(Nota: Hay que dividir entre 2! porque no importa el orden entre los dos pares.)

Segunda forma.   2
13 4
44 = 123 552.
2 2

1.31 Ejemplo. Cuantas manos de pokar tienen corrida?

9
Solucion. El numero mas bajo de la corrida puede ser cualquiera de los siguientes: A,
2, 3, 4, 5, 6, 7, 8, 9 o 10, que son 10 posibilidades. Pongamos 5 casillas; la primera casilla
sera para la carta de numero menor, la siguiente casilla sera para el siguiente numero, y as
sucesivamente hasta la quinta casilla que sera para la carta con el numero mayor. Una vez
escogido el numero menor para la corrida, todos los demas numeros quedan determinados y
lo unico que falta escoger es el palo. Entonces la cantidad de corridas es 1044444 =
10 240.

1.32 Ejercicio. () De cuantas maneras diferentes se pueden ordenar 8 personas


alrededor de una mesa redonda? (Nota: Dos distribuciones se consideraran iguales si una se
puede obtener de la otra mediante un giro.)

1.33 Ejercicio. () De cuantas maneras distintas se pueden sentar 5 personas en una


fila de 8 asientos numerados del 1 al 8?

1.34 Ejercicio. () Cuantas diagonales tiene un polgono regular de n lados?

1.35 Ejercicio. () Probar la Formula de Pascal:


     
n+1 n n
= + ,
r+1 r r+1
para r y n numeros enteros con 0 r < n.

1.36 Ejercicio. El Triangulo de Pascal esta definido como el triangulo de numeros


en el que el renglon numero n aparecen los n + 1 numeros
         
n n n n n
, , , , , .
0 1 2 n1 n
Se muestran a continuacion los primeros 4 renglones del Triangulo de Pascal. Utilizar la
formula del ejercicio anterior para construir los 10 primeros renglones.
1 1
1 2 1
1 3 3 1
1 4 6 4 1
1.37 Ejercicio. De un grupo de 8 jovenes se quiere escoger un equipo de volibol con 6
integrantes y otro de basquetbol con 5 integrantes (deberan tener jugadores en comun) De
cuantas maneras es posible escoger los equipos?

1.38 Ejercicio. () De un grupo de 24 personas se quiere elegir 5 representantes de la


siguiente forma: Pedro y Luis deben estar en el grupo elegido. Hay 8 mujeres en total pero a
lo mas deben figurar 2 en el grupo. De cuantas maneras distintas puede hacerse la eleccion?

10
1.39 Ejercicio. () De un grupo de 30 socios de un club se quiere elegir una mesa
directiva con un presidente, un secretario y 3 equipos de 2 personas cada uno. Cuantas
mesas directivas distintas se pueden formar?

1.40 Ejercicio. () Cuantas palabras distintas se pueden escribir revolviendo las le-
tras de la palabra COMBINATORIA?

1.41 Ejercicio. () De un conjunto de 10 botes de distintos colores se quiere escoger 5


de tal manera que 3 sean para dulces y 2 sean para chocolates. De cuantas formas distintas
es posible hacer la eleccion?

1.42 Ejercicio. () Se dispone de una coleccion de 30 pelotas divididas en 5 tamanos


distintos y 6 colores diferentes de tal manera que en cada tamano hay los 6 colores. Cuantas
colecciones de 4 pelotas tienen exactamente 2 pares de pelotas del mismo tamano (que no
sean las 4 del mismo tamano)?

1.43. El siguiente problema se refiere al conjunto usual de 28 fichas de domino en que


cada ficha muestra dos numeros de la coleccion 0, 1, 2, 3, 4, 5 y 6 (posiblemente repetidos),
como esquematizamos a continuacion:

6 6 6 5 6 4 6 3 6 2 6 1 6

5 5 5 4 5 3 5 2 5 1 5

4 4 4 3 4 2 4 1 4

3 3 3 2 3 1 3

2 2 2 1 2

1 1 1

Se llaman fichas dobles aquellas en que los dos numeros mostrados son iguales. Se llama
mano de domino cualquier coleccion de 7 de las 28 fichas. Notese que el numero total de
manos de domino es 28

7
= 1 184 040.

1.44 Ejercicio. () Cuantas manos de domino tienen por lo menos 2 fichas dobles?

11
2. Teorema del binomio

El siguiente es un resultado muy importante en aritmetica. Lo probaremos aqu utilizando


algunas de las tecnicas de combinatoria que hemos aprendido. Mas adelante volveremos a
probarlo usando el metodo de induccion.

2.1 Teorema. Teorema del Binomio de Newton. Sean a y b numeros arbitrarios


y sea n un numero natural. Entonces
       
n n n n n1 n nr r n n
(a + b) = a + a b + + a b + + b .
0 1 r n
Demostracion. La expresion (a + b)n significa que tenemos que multiplicar a + b consigo
mismo n veces. Entonces, al desarrollar todo el producto, los terminos que obtenemos estan
dados por todas las posibles elecciones de los numeros a o b en cada uno de los n factores
(por ejemplo, (a + b)3 = (a + b)(a + b)(a + b) = aaa + aab + aba + abb + baa + bab + bba + bbb =
a3 + 3a2 b + 3ab2 + b3 ). Observemos entonces que los terminos obtenidos son de la forma as br ,
con 0 s, r n y s + r = n, es decir, s = n r. Ahora notemos que anr br aparece cada
vez que se eligio b en r de los factores y a en el resto, as que el numero de veces que aparece
este termino es nr . Al agrupar terminos semejantes tenemos la formula deseada.

Como hemos visto, los numeros nr (para 0 r n) aparecen como coeficientes en




la expansion de un binomio elevado a la potencia n; por esta razon reciben el nombre de


coeficientes binomiales. En los ejercicios 1.35 y 1.36 vimos que para una n elegida no muy
grande podemos obtener facilmente los coeficientes binomiales sin recurrir en cada caso a la
n n!

formula r = (nr)!r! .

2.2 Ejercicio. Utilizar el Teorema del Binomio para desarrollar la expresion (a 2b)5 .

2.3 Ejercicio. () Utilizar el Teorema del Binomio para desarrollar la expresion (a +


2b 2c )4 .

2.4 Ejercicio. () Encontrar el coeficiente del termino a7 b4 ce2 en el desarrollo de (a +


b + c + d + e)14 . (Sugerencia: Proceder como en la prueba del Teorema del Binomio.)

2.5 Ejercicio. () Utilizar el Teorema del Binomio para probar la formula


       
n n n n
+ + + + = 2n .
0 1 2 n

12
2.6 Ejercicio. () Utilizar el Teorema del Binomio para probar la formula
           
n n n n n n
+ + + = + + .
0 2 4 1 3 5

Que interpretacion se puede dar a esta formula en terminos de subconjuntos de un conjunto?

2.7 Ejercicio. () Probar que para cualquier numero natural se tiene la formula
 2  2  2  2  
n n n n 2n
+ + + + = .
0 1 2 n n

(Sugerencia: Examinar el coeficiente de xn al desarrollar ambos miembros de la igualdad


(1 + x)2n = (1 + x)n (1 + x)n .)

2.8 Ejercicio. () Encontrar el termino que no contiene a x en el desarrollo de


9


1
x +
4x
.

13
3. Comparaciones

En esta seccion veremos algunos ejemplos en los que el contar de dos maneras diferentes
lo mismo nos conduce a obtener ciertas formulas en combinatoria (que a su vez pueden usarse
para resolver otros problemas).

3.1 Ejemplo. Probar la formula de Gauss


n(n + 1)
1 + 2 + 3 + + n = .
2
Solucion. Contaremos las colecciones de 2 elementos que pueden escogerse dentro de
un conjunto de n + 1 elementos de dos maneras diferentes. La comparacion de los dos re-
sultados nos demostrara la veracidad de la formula. Consideremos as el conjunto X =
{x1 , x2 , . . . , xn+1 }. Pongamos los subconjuntos de X que tienen dos elementos en una lista,
como sigue:
{x1 , x2 }, {x1 , x3 }, {x1 , x4 }, {x1 , xn+1 },
{x2 , x3 }, {x2 , x4 }, {x2 , xn+1 },
{x3 , x4 }, {x3 , xn+1 },
..
.
{xn1 , xn+1 }.
De esta lista es facil observar que el numero de subconjuntos de X con 2 elementos es
precisamente lo que aparece del lado izquierdo en la igualdad que queremos probar. Por otro
lado, sabemos que el numero de subconjuntos de 2 elementos que tiene un conjunto con n + 1
elementos es n+1
2
, que es precisamente lo que aparece en el miembro derecho de la igualdad,
y as queda completa la demostracion.

3.2 Ejemplo. Probar la formula


       
n n n n
+ + + + = 2n .
0 1 2 n
Solucion. Probaremos que cada uno de los miembros de la igualdad en la formula cuenta el
numero de subconjuntos que tiene un conjunto de n elementos. Esto es claro para el miembro
izquierdo. Para probar que tambien el miembro derecho representa lo mismo, una vez mas,
contemos otra cosa que es equivalente: el numero de sucesiones de longitud n que se pueden
formar con dos smbolos S y N , es decir el numero de palabras de n letras que pueden
formarse con los smbolos S y N . Obviamente, este numero es 2n , que es el miembro derecho
de la igualdad buscada, as que, para completar nuestra prueba, bastara que probemos que
el numero de sucesiones mencionado es tambien el numero de subconjuntos. Para lograrlo,
establezcamos una correspondencia entre las sucesiones y los subconjuntos de manera que
cada sucesion represente un subconjunto y viceversa. Esto lo podemos hacer observando que
para determinar un subconjunto hay que ir tomando uno a uno los elementos del conjunto
total, e ir diciendo si el elemento pertenece (S) o no (N ) al subconjunto. Para que esto quede

14
mas claro, ilustremos el apareode sucesiones con subconjuntos en el caso del conjunto
X = {a, b, c} (n = 3):
{a, b, c} SSS
{a, b} SSN
{a, c} SN S
{b, c} N SS
{a} SN N
{b} N SN
{c} N N S
{ } N N N.

Es interesante tambien notar que en el ejemplo anterior, al probar que el numero de


subconjuntos de un conjunto con n elementos es 2n , establecimos una comparacion (en este
caso, correspondencia uno a uno) entre los subconjuntos y las sucesiones de longitud n que
constan de los smbolos S y N .

3.3 Ejercicio. Probar la formula de 3.2 usando la forma de contar palabras de cierta
longitud con cierto numero de letras.

3.4 Ejercicio. Probar la formula de Pascal 1.35 utilizando comparaciones.

3.5 Ejercicio. Probar que si n N entonces


       
n1 n n n n
n2 =1 +2 +3 + + n .
1 2 3 n

3.6 Ejercicio. () Probar que si m, n y r son naturales con 0 r m, n, entonces


             
m+n m n m n m n m n
= + + + + .
r 0 r 1 r1 2 r2 r 0

3.7 Ejercicio. () Contar el numero de ternas de numeros del 1 al n (n natural) que


estan en orden creciente de dos maneras distintas para probar la formula
       
n 2 3 n1
= + + + .
3 2 2 2

15
4. Induccion Matematica

En esta seccion recordaremos que significa hacer una demostracion o construccion por
induccion.

4.1 Ejemplo. Analicemos la sucesion (lista) de numeros n2 + n para n natural. El


primer termino de nuestra lista es 2, pues cuando n = 1, n2 + n = 12 + 1 = 2; el segundo
termino es 6 ya que 22 + 2 = 6. As obtenemos la sucesion:

2, 6, 12, 20, 30, 42, 56, 72, 90, . . .

Podemos notar que todos los terminos que escribimos son pares. Es cierto que todos los
terminos de la sucesion son pares?
Solucion. La respuesta es s. Podemos probar esto directamente (sin usar induccion ma-
tematica), observando que para cualquier natural n, el numero n2 + n se puede escribir
como n(n + 1), o sea que todos los terminos de la sucesion son el producto de dos enteros
consecutivos y, como uno de los dos enteros debe ser par, el producto tambien lo sera.
Mas abajo haremos otra demostracion utilizando el metodo de induccion, pero primero
hablemos un poco sobre el procedimiento que seguiremos:
Notemos que con la sola proposicion: Para cualquier natural n, el numero n2 +n es par,
estamos abarcando una infinidad de proposiciones (una para cada n): 12 + 1 es par, 22 + 2
es par, 32 + 3 es par, etc. Si tratamos de probar cada una individualmente no llegaremos
muy lejos; en cambio, si probamos (I1 ) que la primera proposicion es cierta y (I2 ) que,
cada vez que todas las proposiciones anteriores a una fija P sean verdaderas tambien lo es
la misma P, entonces podemos concluir que todas las proposiciones son ciertas. En efecto,
comprobemos por ejemplo que de nuestro metodo (probar (I1 ) e (I2 )) se deduce que la 4a
proposicion es cierta: La 1a proposicion es cierta por (I1 ); utilizando esto tenemos que, por
(I2 ), la 2a proposicion tambien es cierta; pero entonces, al tener que la primera y la segunda
afirmaciones son ciertas, por (I2 ) deducimos que la 3a proposicion es verdadera; ahora ya
tendemos que la primera, la segunda y la tercera son ciertas as que, otra vez usando (I2 )
concluimos que la 4a proposicion tambien es valida. As como llegamos a la 4a proposicion,
a cualquiera podemos llegar en un numero finito de pasos, as que, con solo demostrar (I1 )
e (I2 ), podemos afirmar que todas las proposiciones son ciertas.
Probemos entonces (I1 ) e (I2 ) en nuestro ejemplo.
Demostracion de (I1 ). Tenemos que 12 + 1 = 2, que es par.
Demostracion de (I2 ). Supongamos que k 2 y que todas las afirmaciones desde la pri-
mera hasta la k-esima (es decir, la que se encuentra en el lugar k) son verdaderas. Queremos
utilizar esta suposicion para probar que, en este caso, tambien sera verdadera la (k + 1)-esi-
ma. De hecho en nuestra demostracion utilizaremos solo la validez de la k-esima (es decir, no
requeriremos utilizar toda la fuerza de nuestra suposicion). El que la k-esima afirmacion sea
cierta nos dice que tomamos como verdadero el que k 2 + k sea par y queremos usar esto para
probar que (k + 1)2 + (k + 1) tambien es par. Desarrollemos la expresion (k + 1)2 + (k + 1)

16
para poder compararla con k 2 + k:

(k + 1)2 + (k + 1) = k 2 + 2k + 1 + k + 1 = (k 2 + k) + 2k + 2 = (k 2 + k) + 2(k + 1).

De esta manera hemos logrado expresar (k + 1)2 + (k + 1) como suma de dos numeros pares,
a saber k 2 + k (que estamos suponiendo es par) y 2(k + 1) (que es par por tener el numero
2 como factor). Como la suma de numeros pares tambien es par, (k + 1)2 + (k + 1) es par,
como queramos probar. Esto termina la demostracion de (I2 ).
Concluimos, entonces, que para todo numero natural n, el numero n2 + n es par.

En el metodo de demostracion por induccion se necesita un punto de partida al que se


le llama base de la induccion o, en forma abreviada, (BI). La suposicion de que todas
las proposiciones anteriores a una dada son verdaderas se llama hipotesis de induccion,
abreviado (HI). La base de induccion puede constar de la demostracion de una o de varias
afirmaciones. Tambien la hipotesis de induccion puede constar de la suposicion de que varias
afirmaciones anteriores a la que se va a demostrar sean verdaderas. La practica nos dira que
tan fuerte necesitamos hacer nuestra hipotesis de induccion y cuantas afirmaciones deberan
tomarse como base de induccion.

Una forma de ilustrar por que el metodo de induccion proporciona una demostracion
correcta de algunas proposiciones es la siguiente: Supongamos que se tiene una hilera de
fichas de domino colocadas de manera tal que cada vez que una caiga empujara a la siguiente
para que tambien caiga; si una persona empuja la primera ficha, podremos afirmar que cada
una de las fichas debera caer en algun momento. sin embargo, notemos que es necesario que
las fichas esten lo suficientemente cerca (para que cada una empuje a la siguiente).

La forma en que uno hace ver como la validez de una proposicion (o varias proposiciones)
empuja(n) la validez de la siguiente depende del problema particular de que se trate; a
veces la prueba puede ser muy sencilla y otras muy complicada.

Por otro lado, el que una persona no pueda demostrar satisfactoriamente un resultado
por induccion, no quiere decir nada sobre la validez del resultado; puede ser simplemente que
la sucesion de afirmaciones no tenga una liga tal que la validez de cada afirmacion empuje
la validez de la siguiente. Siguiendo la analoga de las fichas de domino supongamos que
las fichas de domino estuvieran alejadas entre s pero que de todas formas se cayeran por
otra razon (por ejemplo porque colocaramos un ventilador con suficiente fuerza frente a
ellas). Tambien la practica nos dira en que tipo de proposiciones podemos intentar hacer
una demostracion por induccion y en cuales no.

Es importante tambien aclarar que la hipotesis de induccion debe abarcar la base de


induccion; es decir, la primera afirmacion que se suponga verdadera en la hipotesis de induc-
cion debe haber quedado demostrada independientemente en la base. Tambien es importante
hacer notar que en cualquier demostracion por induccion hay un paso comparativo en el
que se establece la relacion o liga que existe entre una afirmacion y la(s) precedente(s).

17
En resumen, para hacer una demostracion por el metodo de induccion matematica se
deberan seguir los siguientes tres pasos:
Primer paso. Identificar la sucesion de proposiciones que abarca la proposicion general
que se va a demostrar.
Segundo paso. Identificar y probar la base de induccion.
Tercer paso. Hacer una hipotesis de induccion (suposicion de que todas las proposiciones
que preceden a una proposicion fija son verdaderas) abarcando la base de induccion, y utilizar
esa suposicion (o parte de ella), para probar que la proposicion fija tambien es cierta. (Para
ello debe haberse hecho una comparacion entre la afirmacion fija que se va a demostrar y
la(s) anterior(es)).

4.2 Ejemplo. Probar que n = 4, 5, 6, . . . implica 2n < n!.


Solucion. La sucesion de proposiciones es:
1a proposicion: 24 < 4!.
2a proposicion: 25 < 5!.
3a proposicion: 26 < 6!.
4a proposicion: 27 < 7!.
..
.

La base de induccion consiste en demostrar la 1a afirmacion. Esto es sencillo ya que


24 = 16, 4! = 24 y 16 < 24, as 24 < 4!.
La hipotesis de induccion puede ser, en este caso, la siguiente: Para cierta k 4 se tiene
k
2 < k!. (Notemos que la primera afirmacion que se toma como cierta en esta hipotesis
es para k = 4 y que esta quedo demostrada en la base de induccion.) Ahora usaremos la
hipotesis de induccion para hacer ver que 2k+1 < (k + 1)!. En efecto, esto se deduce de la
siguiente cadena de igualdades y desigualdades en la que en la primera desigualdad se uso
la hipotesis de induccion y en la segunda desigualdad se utilizo que k + 1 > 2 (esto ultimo
es porque k 4):

2k+1 = 2 2k < 2 k! < (k + 1) k! = (k + 1)!.

(Notemos aqu que las dos igualdades en la cadena fueron de tipo comparativo: sirvieron para
establecer la liga entre la afirmacion que estaba por probarse y la anterior, que se supona
cierta segun la hipotesis de induccion.) Hemos completado satisfactoriamente los tres pasos
en el metodo de induccion, as que el resultado queda probado.

4.3 Ejemplo. Probar por induccion la formula de Gauss

n(n + 1)
1 + 2 + 3 + + n = ,
2
para n natural.

18
Solucion. Notese que el miembro izquierdo de la formula indica que dado el numero
natural n hay que sumar todos los naturales mas chicos que n, incluyendo el mismo n. As,
la sucesion de proposiciones es:
1a proposicion: 1 = 12
2
.
2 proposicion: 1 + 2 = 23
a
2
.
3 proposicion: 1 + 2 + 3 = 34
a
2
.
4a proposicion: 1 + 2 + 3 + 4 = 45
2
.
..
.

En este caso la base de la induccion consiste en demostrar la 1a proposicion, la cual es


obvia.
Tomaremos como hipotesis de induccion la siguiente: Para cierta k 1 (abarcando
la BI) se tiene que 1 + 2 + 3 + + k = k(k+1)
2
. Queremos usar esto para probar que
(k+1) (k+1)+1
1 + 2 + 3 + + (k + 1) = 2
. Para ello tomamos el lado izquierdo de la igualdad
que queremos probar y buscamos la forma de acomodar los terminos para usar la hipotesis
de induccion y despues obtener el lado derecho de la igualdad:

1 + 2 + 3 + + (k + 1) = 1 + 2 + 3 + + k + (k + 1)
= k(k+1)
2
+ (k + 1)
k(k+1)+2(k+1)
. = 2
= (k+2)(k+1)
2
.

Notamos que la primera igualdad es el paso comparativo y en la segunda igualdad se uso la


HI. Esto termina la demostracion.

Con induccion podemos tambien probar formulas en que hay mas de una variable. El
siguiente es un ejemplo tpico. El contenido de la formula es muy util en diversos problemas.

4.4 Ejemplo. Si r es cualquier numero distinto de 1 (no necesariamente natural),


entonces
rn+1 1
1 + r + r2 + + rn = .
r1
para cualquier natural n.

19
Solucion. La sucesion de proposiciones esta indicada por n:
1+1
1a proposicion: 1 + r = r r11 .
r2+1 1
2a proposicion: 1 + r + r2 = r1
.
3+1
3a proposicion: 1 + r + r2 + r3 = r r11 .
4+1
4a proposicion: 1 + r + r2 + r3 + r4 = r r11 .
..
.
Para probar la primera afirmacion (base de la induccion) recordemos que (r + 1)(r 1) =
r2 1 y dividamos esta ecuacion por r 1.
k+1
La HI en este caso es: Para cierta k 1 se cumple 1 + r + r2 + + rk = r r11 . A
partir de esta suposicion probemos la formula correspondiente para n = k + 1:

1 + r + + rk+1 = 1 + r + + rk + rk+1

rk+1 1
= r1
+ rk+1 (por HI)

rk+1 1+(r1)rk+1
= r1

rk+1 1+rk+2 rk+1


= rr1

rk+2 1
= r1

r(k+1)+1 1
= r1
.

De esta serie de igualdades concluimos que, si la formula se supone valida para n = k,


entonces tambien lo sera para n = k + 1, y con esto completamos satisfactoriamente todos
los pasos en el metodo de induccion. Concluimos entonces que la formula es cierta para todo
n.

4.5 Ejercicio. Probar la afirmacion del ejemplo anterior en forma no inductiva. (Su-
gerencia: Utilizar la misma idea con la que probamos la base de induccion.)

4.6 Ejercicio. Hacer una prueba inductiva y otra no inductiva de que el producto de
tres enteros consecutivos es multiplo de 6.

4.7 Ejercicio. () Hacer una prueba inductiva y otra no inductiva de la siguiente


formula para n natural:
1 1 1 n
+ + + = .
12 23 n (n + 1) n+1

20
1 1 1
(Sugerencia: Para la prueba no inductiva, observar que k(k+1)
= k
k+1
.)

En los siguientes ejemplos compararemos algunas definiciones no recursivas con otras


recursivas.

4.8 Ejemplo. Definamos la sucesion a1 , a2 , a3 , . . . recursivamente por a1 = 1 y, para


n 2, an = an1 + 2. Encontrar los primeros 6 terminos de la sucesion y dar una definicion
no recursiva de ella.
Solucion. Para obtener los primeros 6 terminos de la sucesion partimos de la base a1 = 1
y vamos construyendo los siguientes terminos sumando 2 al termino recien construido: 1, 3,
5, 7, 9, 11. Una definicion no recursiva de la misma sucesion es an = 2n 1.

El ejemplo anterior es un caso particular de las llamadas sucesiones o progresiones


aritmeticas; en general una sucesion aritmetica es una sucesion de numeros a1 , a2 , a3 , . . .
en que la diferencia entre dos terminos consecutivos cualesquiera es un numero constante d,
es decir an+1 = an + d, para toda n.
Otros ejemplos de sucesiones aritmeticas son:
1, 2, 3, 4, 5, . . . (aqu d = 1 y a1 = 1),
2, 4, 6, 8, 10, . . . (aqu d = 2 y a1 = 2),
10, 17, 24, 31, 38, . . . (aqu d = 7 y a1 = 10).
0, 21 , 1, 23 , 2, 52 , . . . (aqu d = 1
2
y a1 = 0).

Hemos definido sucesion aritmetica por recursion, es decir, en forma inductiva; si solo
contamos con esto, es de esperar que cualquier afirmacion sobre una sucesion aritmetica
utilice induccion; el siguiente ejemplo (demostrado por induccion), permitira trabajarlas en
forma no recursiva; en particular el resultado nos dice como conocer cualquier termino de la
sucesion sin necesidad de conocer el anterior.

4.9 Ejemplo. Sea a1 , a2 , a3 , . . . una sucesion aritmetica con diferencia d (es decir, para
toda n, an+1 = an + d. Probar que para n 2 se tiene an = a1 + (n 1)d.
Solucion. La sucesion de afirmaciones es:
1a afirmacion: a2 = a1 + (2 1)d.
2a afirmacion: a3 = a1 + (3 1)d.
3a afirmacion: a4 = a1 + (4 1)d.
4a afirmacion: a5 = a1 + (5 1)d.
..
.
La base de induccion es, en este caso, la primera afirmacion (es decir, la afirmacion
para n = 2). Es facil darse cuenta de la validez de esta proposicion pues, por definicion,
a2 = a1 + d. Hagamos ahora la hipotesis de induccion: Para cierta k 2 es verdad que
ak = a1 + (k 1)d. Utilizando esta HI probemos que tambien es cierto el resultado para

21
n = k + 1:
ak+1 = ak + d (por definicion)
= a1 + (k 1)d + d (por HI)
= a1 + kd.
Esto termina la demostracion.

4.10 Ejercicio. () Una sucesion o progresion geometrica con razon r es una sucesion
de numeros a1 ,a2 ,. . . en que cada uno se obtiene del anterior multiplicando por el numero r (es
decir, para n 2, an = an1 r). Dar una definicion no recursiva para la sucesion geometrica
con a1 = 5 y r = 21 . Calcular la suma de los primeros 100 terminos de esta sucesion.

En todas las pruebas por induccion que hemos hecho hasta el momento, al demostrar
que la (k + 1)-esima afirmacion es verdadera solo hemos utilizado la validez de la k-esima
afirmacion; inclusive, en cada caso simplificamos la hipotesis de induccion de tal manera que
abarcara solo la afirmacion anterior a la que queramos probar y no todas las anteriores. En
los ejemplos que trataremos a continuacion s necesitaremos hacer la hipotesis de induccion
como la anunciamos al principio de esta seccion. La diferencia entre los casos que siguen y
los anteriores es que cada afirmacion esta ligada no solo a la que la precede sino a una o
mas de las anteriores. La practica nos dira como reconocer en que caso nos encontramos;
mientras tanto, podemos siempre hacer la hipotesis de induccion en su forma mas general
y, una vez que estemos en el tercer paso de la demostracion inductiva, utilizar solo lo que
necesitemos de la hipotesis.

Considerando que llegado este punto ya debe ser obvio para el lector el primer paso de la
induccion (es decir, identificar la sucesion de afirmaciones que abarca la afirmacion general
que se quiere probar), de aqu en adelante ya no incluiremos este en nuestras demostraciones.

4.11 Ejercicio. Probar por induccion que para n natural se tiene la formula:
n(n + 1)(2n + 1)
12 + 22 + 32 + + n2 = .
6

4.12 Ejercicio. Probar la formula del ejercicio anterior en forma no inductiva como
sigue: Para k = 1, 2, . . . sea ak = (k + 1)3 k 3 . Calcular la suma a1 + a2 + + an de dos
maneras distintas e igualar. Despejar 12 + 22 + 32 + + n2 .

4.13 Ejercicio. () Calcular directamente la suma 13 +23 +33 + +n3 para n = 1, 2, 3


y 4; usar esto para proponer una formula para calcular la suma para cualquier natural n, y
probar la formula por induccion.

4.14 Ejercicio. Calcular la suma 13 + 23 + 33 + + n3 utilizando un razonamiento


parecido al del ejercicio 4.12. (Sugerencia: En este caso debera considerarse ak = (k +1)4 k 4
para k = 1, 2, . . ..)

22
4.15 Ejercicio. () Calcular la suma

1 1000 + 2 999 + 3 998 + + 999 2 + 1000 1.

4.16 Ejercicio. () Calcular la suma


1 1 1 1
+ + + + .
13 35 57 999 1001

4.17 Ejercicio. () Sea a0 ,a1 ,a2 ,. . . la sucesion de numeros definida recursivamente


como sigue: a0 = 1, a1 = 1 y, para n 2, an = an1 +a 2
n2
. Probar por induccion que para
n 1, an 0.

Haremos a continuacion algunos ejemplos en que la demostracion por induccion tiene


algunos puntos mas delicados.

4.18 Ejemplo. La sucesion de Fibonacci f1 , f2 , f3 , . . . se define como sigue: f1 = 1,


f2 = 1 y, para n 3, fn = fn1 + fn2 . Construir los primeros 10 terminos de la sucesion y
probar la siguiente formula que nos proporciona una definicion no recursiva de la sucesion:
 n  n
1+ 5
2
12 5
fn = .
5
Solucion. Construyamos los primeros 10 terminos de la sucesion siguiendo la definicion:
Los primeros dos terminos son ambos 1 y, para construir cada uno de los terminos siguientes,
sumemos cada vez los ultimos dos que ya tengamos: 1,1,2,3,5,8,13,21,34, 55. Como pudimos
observar en la misma definicion, para conocer un termino es necesario conocer no solo el
inmediato anterior sino los dos que le preceden. Es natural entonces pensar que una demos-
tracion por induccion de una afirmacion sobre todos los terminos de la sucesion de Fibonacci
deba tener una hipotesis de induccion que abarque las afirmaciones correspondientes a los
dos terminos que preceden al que se considera en ese momento. Por otro lado, los primeros
dos terminos estan dados de manera independiente y los demas se basan en ellos; por esta
razon, la base de induccion debe constar de la prueba de las dos afirmaciones correspon-
dientes a estos terminos. Tomemos el lado derecho de la formula que queremos probar para
n = 1:  1  1
1+ 5
12 5
2 (1 + 5) (1 5) 2 5
= = = 1 = f1 .
5 2 5 2 5
Hagamos ahora lo mismo para n = 2:
 2  2
1+ 5
12 5
2 (1 + 2 5 + 5) (1 2 5 + 5) 4 5
= = = 1,
5 4 5 4 5

23
que es igual a f2 . Con esto concluimos la base de induccion. Ahora tomemos k 3 y hagamos
la hipotesis de induccion: La formula es verdadera para todos los naturales menores que k.
Tenemos entonces

fk = fk1 + fk2 (por definicion)


 k1  k1  k2  k2
1+ 5 1 5 1+ 5 1 5
2
2 2
2
= + (por HI)
5 5
 k2 
  k2  
1+ 5 1+ 5
2
+ 1 12 5
2
1 5
2
+ 1
=
5
 k2    k2  
1+ 5 3+ 5
2 2
12 5 3 5
2
=
5
Por otro lado, consideremos el miembro derecho de la formula para n = k:
 k  k  k2  2  k2  2
1+ 5
2
12 5 1+ 5
2
1+ 5
2
12 5 1 5
2
=
5 5
 k2    k2  
1+ 5 1+2 5+5
2 4
12 5 12 5+5
4
=
5
 k2    k2  
1+ 5 3+ 5
2 2
12 5 3 5
2
= .
5
Hemos obtenido lo mismo que tenamos arriba, as que la formula tambien es verdadera para
n = k, y esto concluye la demostracion.

4.19 Ejemplo. Definamos una sucesion a0 ,a1 ,a2 ,. . . como sigue: a0 = 1 y, para n 1,
       
n n n n
an = a0 + a1 + a2 + + an1 .
0 1 2 n1

Probar que todos los terminos de la sucesion son impares.


Solucion. Antes de empezar la demostracion de que todos los terminos son impares no-
temos primero que en la misma definicion de la sucesion se hizo una recursion que utiliza no
solo el termino anterior al que se esta definiendo sino todos los anteriores. Es natural entonces
pensar que para probar que un termino an (n 1) es impar, debemos utilizar el que todos
los anteriores (a0 ,a1 ,a2 ,. . .,an1 ) lo son; as que en este caso, la hipotesis de induccion debera
abarcar todos estos. Conviene tambien escribir los primeros terminos de la sucesion, pues el
analisis cuidadoso de varios terminos en particular muchas veces da una idea de como hacer
la demostracion general.

24
Tenemos:

a0 = 1,
 
1
a1 = 1
0
= 1 1 = 1,
   
2 2
a2 = 1+ 1
0 1
= 1 1 + 2 1 = 3,
     
3 3 3
a3 = 1+ 1+ 3
0 1 2
= 1 1 + 3 1 + 3 3 = 13,
       
4 4 4 4
a4 = 1+ 1+ 3+ 13
0 1 2 3
= 1 1 + 4 1 + 6 3 + 4 13 = 75,
         
5 5 5 5 5
a5 = 1+ 1+ 3+ 13 + 75
0 1 2 3 4
= 1 1 + 5 1 + 10 3 + 10 13 + 5 75 = 541.

Observamos aqu que los coeficientes que van apareciendo son los del triangulo de
 Pascal, el
cual sabemos que es simetrico respecto a la vertical central (esto es, nr = nr n

). Tambien
sabemos que los terminos centrales en los renglones pares (es decir, los de la forma nn ) son

2
todos numeros pares (pues son la suma de los dos numeros iguales arriba de el). Hechas estas
observaciones procedamos con la demostracion.
La base de induccion es la prueba de que el primer termino de la sucesion (es decir, a0 )
es impar, lo cual es cierto por definicion. Tomemos k 1 y supongamos que a0 ,a1 ,a2 ,. . .,ak1
son impares (esta es nuestra hipotesis de induccion). Probaremos que ak es impar. Dividimos
la prueba en dos casos: cuando k es impar y cuando k es par. En el primer caso, factorizando
los coeficientes binomiales con sus simetricos, tenemos
     
k k k 
ak = 1 + (a1 + ak1 ) + (a2 + ak2 ) + + k1 a k1 + a k+1 .
1 2 2
2 2

Ahora utilizamos la hipotesis de induccion: como cada ai (con 1 i k 1) es impar, cada


una de las sumas a1 + ak1 , a2 + ak2 , . . ., a k1 + a k+1 es un numero par; con esto ya es claro
2 2
que ak es impar, y aqu termina la prueba para el caso en que k sea impar. En el caso en
que k sea par, agrupamos de la misma manera pero nos sobrara un termino sin agrupar:
     
k k k
ak = 1 + (a1 + ak1 ) + (a2 + ak2 ) + + k a k .
1 2 2
2

25
Sin embargo, el termino no agrupado tambien es par pues el coeficiente binomial en el lo es.
Esto concluye la prueba en el caso en que k es par. Hemos completado satisfactoriamente
los pasos de la induccion en todos los casos.

4.20 Proposicion. Todo conjunto con n elementos tiene 2n subconjuntos


Demostracion. El resultado es obvio para cuando n = 0 pues el conjunto con 0 elementos
solo tiene un subconjunto que es el mismo. Sea n 1; HI: Todo conjunto con n1 elementos
tiene 2n1 subconjuntos. Consideremos el conjunto X = {x1 , x2 , . . . , xn } con n elementos.
Queremos utilizar la HI para probar que X tiene 2n subconjuntos. Consideremos el conjunto
Y obtenido al quitarle a X el elemento xn . Por HI, Y tiene 2n1 subconjuntos. Ahora bien,
los subconjuntos de X podemos dividirlos en dos clases: los que no tienen al elemento xn
(es decir, los que estan contenidos en Y ) y los que s lo tienen. El numero de conjuntos de
las dos clases es el mismo pues cada conjunto de la segunda clase se obtiene adjuntando el
elemento xn a uno de los conjuntos de la primera. Entonces, por HI, cada una de estas clases
tiene 2n1 conjuntos; en total X tendra 2n1 + 2n1 = 2 2n1 = 2n subconjuntos, como
queramos probar. Esto termina la demostracion.

4.21 Ejercicio. Sea X = {x1 , x2 , x3 , x4 }. Encontrar las dos clases de subconjuntos de


X de que se habla en la demostracion de 4.20 y aparear los conjuntos de una clase con los
de la otra como indica esa prueba.

4.22 Ejercicio. () Demostrar que todo conjunto tiene la misma cantidad de subcon-
juntos con un numero par de elementos que con un numero impar.

4.23 Ejercicio. () Si a1 = 1 y, para n 2, an = n + (1)n an1 , cuanto valen a1000 ,


a2001 , a3002 , a4003 ?

4.24 Ejercicio. () Probar por induccion la formula siguiente para n natural:


     
n n n
+ + + = 2n .
0 1 n

(Sugerencia: Utilizar la Formula de Pascal.)

4.25 Ejercicio. () Probar por induccion la formula siguiente para n y k enteros con
0 k n:          
k k+1 k+2 n n+1
+ + + + = .
k k k k k+1
(Sugerencia: Usar la Formula de Pascal.)

4.26 Ejercicio. () La siguiente afirmacion es obviamente falsa: En cualquier lista


de n numeros, todos son iguales entre s. Determinar cual es el error en la demostracion

26
por induccion que presentamos a continuacion (es decir, encontrar en que momento el pro-
cedimiento que se sigue en la supuesta demostracion es incompleto o incorrecto): BI: Para
n = 1 la afirmacion es verdadera pues solo hay un numero en la lista. HI: Supongamos que el
resultado es cierto para cualquier lista de n numeros y tomemos una lista de n + 1 numeros:
a1 , a2 , . . . , an , an+1 . Entonces, por HI, los primeros n numeros a1 , a2 . . . , an son iguales entre
s; aplicando tambien la hipotesis de induccion a los ultimos n numeros: a2 , . . . , an , an+1 ,
estos son tambien iguales entre s. Pero entonces todos son iguales a a2 , as que todos son
iguales entre s.

4.27 Ejercicio. () La siguiente afirmacion es obviamente falsa: Para construir una


red carretera en un pas de manera que cualesquiera dos ciudades esten conectadas mediante
la red, es suficiente que a cada ciudad llegue al menos una carretera (entendiendo que las
carreteras sean todas de doble sentido). Determinar cual es el error en la prueba por
induccion que presentamos a continuacion: Para dos ciudades el resultado es claramente
cierto. Supongamos por induccion que un pas con n ciudades cumple la propiedad y agre-
guemos una ciudad C; por hipotesis, de C sale una carretera, digamos a D. Entonces, a traves
de D, la ciudad C esta conectada con las demas, y as, todas las ciudades estan conectadas
entre s.

4.28 Ejercicio. () La siguiente afirmacion es obviamente falsa: El conjunto N de los


numeros naturales es finito. Determinar cual es el error en la prueba por induccion que
presentamos a continuacion: Para cada natural n sea An = {n}. Sabemos que la union de
todos los conjuntos An nos da el conjunto N, y que la union de dos conjuntos finitos es finito.
Entonces BI: A1 A2 es finito. HI: Supongamos que A1 A2 An1 es finito para
cierta n 3. Entonces, A1 A2 An = (A1 A2 An1 ) An , que es la union
de dos conjuntos finitos (usando la HI), as que A1 A2 An tambien es finito. En
conclusion, N es finito.

27
5. Caminos

Analizaremos en esta seccion un ejemplo basico de cuenta de caminos siguiendo lneas en


figuras. En este ejemplo aprenderemos dos tecnicas para la resolucion de problemas de este
estilo y tendremos oportunidad de practicar algunas tecnicas de combinatoria aprendidas
antes.

5.1 Ejemplo. En una cuadrcula de m por n, sea A el vertice inferior izquierdo y sea
B el vertice superior derecho. Cua ntos caminos hay de A a B siguiendo las lneas de la
figura, si solo se puede avanzar hacia la derecha y hacia arriba?
Solucion. Ilustremos un camino con m = 4 y n = 6.

B
......................................
........
......
......
......
............................................
.....
.....
......
......
.....
............................................................................................
......
......
......
......
......
......
......
......
......
......
........................................
A
Primera forma. Notemos que cada camino debe recorrer una distancia total de m + n =
10 segmentos, de los cuales 6 segmentos se recorren horizontalmente y 4 verticalmente.
Entonces cada camino se puede identificar con una palabra de 10 letras que usa 6 letras
H (horizontal) y 4 letras V (vertical). (Por ejemplo, el camino de la figura esta representado
por la palabra HV V HHHV HV H.) Como sabemos, el numero de estas palabras es 10

4
.

Segunda forma. Observemos que hay dos maneras para llegar a cada vertice que no forme
parte del lado izquierdo o del lado de abajo de la cuadrcula (a los cuales solo hay una forma
de llegar): verticalmente (desde el vertice inmediatamente abajo de el) y horizontalmente
(desde el vertice inmediatamente a la izquierda de el). Entonces, el numero de caminos que
llegan a uno de esos vertices es la suma de los caminos que llegan a los vertices adyacentes
a la izquierda y abajo de el. As podemos poner en la figura, junto a cada vertice, el numero
de caminos que llegan a el:

28
1 5 15 35 70 126 210
B
1 4 10 20 35 56 84
1 3 6 10 15 21 28
1 2 3 4 5 6 7
1 1 1 1 1 1 1
A

5.2 Ejercicio. () Dentro de un cubo de alambre C de dimensiones 5 5 5 se colocan


alambres dividiendo C en cubos de dimensiones 1 1 1. Llamese A al vertice inferior
izquierdo de la cara anterior de C y sea B el vertice opuesto a A en C (es decir, B es el
vertice superior derecho de la cara posterior de C). Cuantos caminos diferentes llegan del
punto A al punto B siguiendo los alambres del cubo, si las unicas direcciones permitidas son:
hacia atras, hacia la derecha y hacia arriba?

5.3 Ejercicio. () Usar caminos para probar la formula de naturales n:


 2  2  2  2  
n n n n 2n
+ + + + = .
0 1 2 n n

(Sugerencia: Contar caminos en una cuadrcula de n n como en la primera forma en el


ejemplo 5.1, fijandose en los puntos donde el camino atraviesa la diagonal que va de la esquina
superior izquierda a la esquina inferior derecha.)

5.4 Ejercicio. Usar caminos para probar la formula de Pascal


     
n+1 n n
= + ,
r+1 r r+1

para r y n numeros enteros con 0 r < n. (Sugerencia: Comparar las dos formas de resolver
el ejemplo 5.1.)

5.5 Ejercicio. () Siguiendo las lneas de la figura, cuantos caminos hay para ir del
punto A al punto B que no pasen dos veces por el mismo punto y que solo avancen hacia
abajo y hacia los lados pero no hacia arriba? [3er Examen Nacional]

29
A ..
......
... .....
... ...
.... ...
.
...................................
... ..... .
. .. .. ..... .....
.
.. ... .. ...
... . .... . ...
.....................................................................
... ... . .. . .
.... ..... .... .... .... .....
.. ... ... ... ... ...
... .. . .. . .
.................................................................................................
...... .. .... .. ..... ..
... .....
. .. .. .... .....
..
. . . .... .... .... .... ... ...
... ... .. ... ... ... ... ...
. . .. . . . .
................................................................................................................................
.........
. ... ... ... ... ... ... ... ...
.
.... ..... .... ..... .... ..... .... .... .... .....
. ... ... ... ... ... ... ... ... ...
... .. . .. .. .
.................................................................................................................................................................
... ... .. .. .. .. .. . .. . .. ..
... .... .... ..... .... ..... .... ..... .... ..... .... .....
.
..
. ... ... .. ... .. ... ... ... ... ... ...
. ...... ...... ...... ...... ..... ...
... .. .. .. .. ..
..................................................................................................................................................................................................
... ....
. . ... . ... .. ... .. ... .. .. .. .
.
...
... ..
. .
.. ... ..
. .
. . ... ... . ... ... . ... ... .... ... .....
. .
. ... ... ... ... ... .. . ... .. . ... .. . ... .. . ...
..
....................................................................................................................................................................................................
B

5.6 Ejercicio. () Cuantos caminos hay del punto A al punto B siguiendo las lneas
de la figura si las direcciones permitidas son , %, &, ., - (es decir, cualquier sentido esta
permitido salvo ) y no se permite pasar dos veces por el mismo punto?
........................... ........................... ........................... ...........................
... ...
..
.. ... ..
..
... ..
..
...
.. ...
..........................
...
..........................
...
..........................
...
...
. ....
.. ...
.. ...
.. ..... . ... . ... . ...
......................
. ...
... ... ..
. . ..
. . ..
. . .. ...
.
.. ...
. ... ...
. ... ...
. ... ...
. .... ...
.. ... .. ... .. ... .. ... .. ...
A ..
...
...
...
... .. ....
.
.. ..
......................
...
.... ..
.
. ..
.
.. ..
......................
...
.... ..
.
...
.
.. ..
......................
...
.... ..
.
...
.
.. ..
......................
...
....
B
.
..
..
...
.. .
........................... .
........................ .. .
........................ .. ......................... .
.
........................ . ... . . . . .
... ... ...
... ... ... ... ... ... ... ...
... ... ... ... ... ... ... ...
.......................... ...
.....................
.. .......................... ..........................

30
6. Inclusion y Exclusion

Empecemos esta seccion analizando un ejemplo que nos dara la clave del llamado Prin-
cipio de Inclusion y Exclusion.

6.1 Ejemplo. Cuantos numeros menores que 10 000 no son divisibles ni por 2, ni por
3, ni por 5?
Solucion. A 10 000 habra que restarle la cantidad de numeros divisibles por alguno de 2,
3 o 5. Sin embargo esto hay que hacerlo con cuidado para evitar repeticiones; por ejemplo,
los numeros que son divisibles tanto por 2 como por 3 se consideran dos veces: al contar los
divisibles por 2 y al contar los divisibles por 3. Vamos a determinar primero, por separado,
cuantos multiplos hay de cada una de las distintas combinaciones entre 2, 3 y 5. Hay:

5 000 numeros divisibles por 2,


3 333 divisibles por 3,
2 000 divisibles por 5,
1 666 divisibles por 6,
1 000 divisibles por 10,
666 divisibles por 15 y
333 divisibles por 30.

Al restarle a 10 000 la cantidad de numeros divisibles por 2 y luego los divisibles por 3 y a
continuacion los divisibles por 5:

10 000 (5 000 + 3 333 + 2 000),

los que son divisibles por 6, por 10 o por 15 pero no por 30 se habran quitado dos veces
cada uno, y los que son multiplos de 30 se habran quitado tres veces. Entonces al agregar a
la cuenta los que son multiplos de 6, de 10 o de 15, los que son divisibles por 30 se habran
quitado primero tres veces al restar los multiplos de 2, de 3, y de 5, y despues se habran
vuelto a sumar tres veces al sumar los multiplos de 6 y los de 10 y los de 15, as que tendremos
que restarlos. La respuesta al ejemplo es pues:

10 000 (5 000 + 3 333 + 2 000) + (1 666 + 1 000 + 666) 333 = 2 666.

6.2 Ejercicio. En cierta escuela hay 100 alumnos. De ellos 50 saben ingles, 30 saben
aleman y 30 saben frances. Ademas 10 saben ingles y frances, 14 saben frances y aleman,
11 saben ingles y aleman, y 6 saben los tres idiomas. Determinar cuantos alumnos no saben
ninguno de los tres idiomas.

El procedimiento que utilizamos en el ejemplo anterior puede generalizarse, segun veremos


en el siguiente principio.

31
6.3 Proposicion. Principio de Inclusion y Exclusion. Supongamos que tenemos n
conjuntos A1 , A2 , . . . , An (posiblemente con elementos en comun). Entonces el numero total
k de elementos que tienen entre todos es igual a k1 k2 + k3 k4 + kn , donde k1 es la
suma de los elementos que pertenecen a (por lo menos) uno de los conjuntos, k2 es la suma
de los elementos que pertenecen a (por lo menos) dos de los conjuntos, y as sucesivamente
hasta kn , que es el numero de elementos en comun a todos los conjuntos. (Utilizando el
lenguaje usual de teora de conjuntos donde |X| denota el numero de elementos de un
conjunto X, es el smbolo usual de union y es el smbolo usual de interseccion, tenemos:
k = |A1 A2 An |, k1 = |A1 | + |A2 | + + |An |, k2 = |A1 A2 | + |A1 A3 | + + |A1
An | + |A2 A3 | + + |An1 An |, y as sucesivamente hasta kn = |A1 A2 An |.)
Demostracion. Tomemos un elemento cualquiera y supongamos, por ejemplo, que el ele-
mento pertenece a los conjuntos Ai1 , Ai2 , . . . , Air para cierta r, y solo a estos. Entonces el
numero de veces que dicho elemento se considera en la suma k1 k2 + k3 k4 + kn es
         
r r r r r
+ + ,
1 2 3 4 r
que, por el ejercicio 4.22, es igual a 0r , que es 1. Entonces la suma k1 k2 + k3 k4 + kn


cuenta cada elemento exactamente una vez, que es lo que queramos demostrar.

Para el siguiente ejemplo recordemos que todo numero natural N se puede escribir en
forma unica como producto de potencias de primos p1 < p2 < < pr : N = pa11 pa22 par r
(a1 , a2 , . . . , ar son naturales) y que a esta se le llama la descomposicion canonica del
numero N . Recordemos tambien que se dice que dos numeros son primos relativos cuando
su maximo comun divisor es 1 (en otras palabras, cuando los primos que aparecen en sus
descomposiciones canonicas, son distintos).

6.4 Ejemplo. Sea N = pa11 pa22 par r la descomposicion canonica del numero natural
N . Se llama (N ), o funcion de Euler al numero de enteros menores que N y primos
relativos con N . (Por ejemplo (12) = 4 pues los enteros menores que 12 y primos relativos
con 12 son 4, a saber, 1, 5, 7 y 11.) Probar que
(N ) = pa11 1 pa22 1 par r 1 (p1 1)(p2 1) (pr 1).
Solucion. Aplicamos el Principio de Inclusion y Exclusion. Los multiplos de cada pi son
N
pi
los multiplos de cada pi pj (para i 6= j) son pN
; i pj
; los multiplos de los productos pi pj pk
(para i, j y k ndices distintos) son pi pNj pk , y as sucesivamente. Entonces (N ) es igual a:

N ( pN1 + + N
pr
) + ( p1Np2 + + N
pr1 pr
) + (1)r p1 pN
2 pr
=

pa11 pa22 par r (pa11 1 pa22 par r + pa11 pa22 1 par r + + pa11 pa22 par r 1 )+

a 1
(pa11 1 pa22 1 par r + + pa11 pr1
r1
par r 1 ) + +

(1)r pa11 1 pa22 1 par r 1 .

32
Podemos observar que los sumandos en la expresion anterior son todas las expresiones de la
forma pb11 pb22 pbrr , donde cada bi es igual a ai o a ai 1; ademas la paridad del numero de
sumandos que son ai 1 determina el que se sume o se reste el termino correspondiente en
la expresion. Entonces la expresion se puede simplificar:

(N ) = (pa11 pa11 1 )(pa22 pa22 1 ) (par r par r 1 )


= pa11 1 (p1 1)pa22 1 (p2 1) par r 1 (pr 1),

que es la formula que queramos probar.

6.5 Ejercicio. Usar la formula probada en el ejemplo anterior para calcular (600),
(1995), (23) y (128).

6.6 Ejercicio. () Se dice que una mano de domino tiene falla si alguno de los numeros
entre el 0 y el 6 no aparece en la mano (cada numero faltante es una falla); por ejemplo la
mano

2 1 , 5 5 , 3 1 , , 1 , 6 5 , 2

tiene falla a 40 s. Probar que el numero de manos de domino que no tienen falla es 501 015.

6.7 Ejercicio. () En una oficina hay 10 empleados. Cada uno es especialista en una
labor distinta a la de los demas. Para no aburrirse, les gusta intercambiar sus puestos; sin
embargo, el buen funcionamiento de la oficina exige que en cada momento haya exactamente
4 empleados trabajando en su especialidad. Cuantas distribuciones de los puestos se pueden
hacer bajo estas condiciones?

33
7. Probabilidad Combinatoria

Intuitivamente, la probabilidad calcula la proporcion de casos en los que cierto experi-


mento ocurre en relacion con el total de resultados posibles.

El conjunto de todos los resultados posibles de un experimento se llama espacio mues-


tral. Cualquier subconjunto del espacio muestral se llama suceso.

Analicemos algunos ejemplos en los que el espacio muestral es finito y, en ese caso,
definamos la probabilidad de que ocurra un suceso S, en smbolos P (S), como
|S|
P (S) = .
||
(Aqu se esta suponiendo que todos los resultados del experimento tienen la misma proba-
bilidad de ocurrir; mas adelante se veran ejemplos de distinta ndole.)

Se piensa entonces que la probabilidad es una funcion que va del conjunto de sucesos al
conjunto de los numeros racionales. El valor de un suceso es 0 cuando no puede ocurrir y es
1 cuando es seguro que ocurre.

En muchos de nuestros problemas aparece el conunto de los numeros naturales del 1 al


n. Para simplificar, denotaremos a este conjunto por n, es decir,

[n] = {1, 2, . . . , n}.

7.1 Ejemplo. El experimento consiste en lanzar un dado y observar el numero que


queda arriba. Calcular la probabilidad de que el numero que quede arriba sea el 1 y tambien
calcular la probabilidad de que el numero que quede arriba sea par.
Solucion. Aqu M = [6]. En el primer caso el suceso es S = {1} y P (S) = 61 . En el
segundo caso el suceso es T = {2, 4, 6} y P (T ) = 36 = 12 .

7.2 Ejemplo. El experimento es lanzar una moneda 2 veces y observar la sucesion de


aguilas a y soles s que se obtiene. Determinar la probabilidad de que se observen dos aguilas.
Solucion. El espacio muestral puede ser

= {aa, as, sa, ss}

y entonces el suceso es S = {aa} y P (S) = 41 .

7.3 Ejemplo. Calcular la probabilidad de que al lanzar una moneda 3 veces se mues-
tren al menos dos aguilas.
Solucion. Aqu podemos definir

= {aaa, aas, asa, saa, ass, sas, ssa, sss}.

34
El suceso es S = {aas, asa, saa, aaa} y entonces la probabilidad buscada es P (S) = { 48 =
1
2
.

7.4 Ejemplo. Determinar la probabilidad de que al lanzar dos dados lo que sumen las
caras que se ven arriba sea 6.
Solucion. Conviene definir = [6] [6] y entonces
S = {(1, 5), (2, 4), (3, 3), (4, 2), (5, 1)},
5
de donde la probabilidad es 36
0.14.

Veamos algunas propiedades que ya hemos podido observar en los ejemplos y hagamos
algunos comentarios sobre ellas.

De aqu en adelante denota al espacio muestral en cuestion.

7.5 Nota. La probabilidad de que algo ocurra es un numero entre 0 y 1. (Esto es obvio
pues, como el suceso S es subconjunto del espacio muestral , entonces |S| |M |.) Es 0
cuando es imposible que ocurra (es decir, P () = 0), y es 1 cuando es seguro que debe ocurrir
(o sea, P (M ) = 1). En el caso en que es conjunto finito, para todo S se tiene que
P (S) Q.

7.6 Nota. Si dos cosas no pueden ocurrir simultaneamente, la probabilidad de que


ocurra una o la otra (es decir, cualquiera de las dos) es la suma de las probabilidades. En otras
palabras, si S y T son sucesos ajenos (es decir, S T = ), entonces P (S T ) = P (S)+P (T ).
(Esto es claro pues |S T | = |S| + |T |.)

Retomemos aqu el ejemplo arriba en el que se pide calcular la probabilidad de que al


lanzar tres monedas al aire salgan al menos dos aguilas. Arriba calculamos la probabilidad
de S T donde S = {ass, sas, ssa, } y T = {aaa}, pero podramos haber calculado por
separado las probabilidades P (S) = 83 y P (T ) = 81 .

Observemos que la propiedad anterior no sera valida si no pidieramos que los suce-
sos fueran mutuamente excluyentes, es decir, si hubiera la posibilidad de que ocurrieran
simultaneamente; por ejemplo, la probabilidad de que al lanzar un dado lo que salga sea un
numero mayor que 3 o que sea un numero par es 46 (el suceso es {2, 4, 5, 6}) y no 63 + 36 = 1,
que sera la suma de las probabilidades de los sucesos S = {4, 5, 6} y T = {2, 4, 6} (los casos
4 y 6 son comunes a los dos y se estaran contando dos veces al sumar las probabilidades).

7.7 Nota. La probabilidad de que ocurran dos cosas en un orden determinado es el


producto de las probabilidades. En este caso estamos diciendo que si S1 es un suceso en un
espacio muestral 1 y S2 es un suceso en un espacio muestral 2 , entonces P (S1 S2 ) =
P (S1 )P (S2 ), lo cual es claro pues dados dos conjuntos A y B, el numero de elementos del
producto cartesiano A B es |A||B|.

35
Retomemos el ejemplo arriba en el que queramos calcular la probabilidad de que al
lanzar dos monedas al aire el resultado en ambas sea aguila. En lugar de lo hecho arriba,
podramos haber definido = {a, s} y S = {a} y calcular P (S)P (S) = 21 12 = 14 .

Dado S denotamos por S al complemento de S, es decir, al conjunto \ S = {x


:x/ S}

7.8 Corolario. Si la probabilidad de que algo ocurra es p, entonces la probabilidad de


que no ocurra es 1 p.
Demostracion. Esto es claro por la propiedad (2) pues para S , S y S son conjuntos
ajenos cuya union es , as que 1 = P (M ) = P (S) + P (S).

Veamos mas ejemplos en los que podremos observar que hay que escoger con cuidado el
espacio muestral para que represente verdaderamente el problema que se quiere resolver.

7.9 Ejemplo. El experimento es sacar 2 pelotas de una caja en la que hay 2 pelotas
rojas y 3 azules. Se quiere calcular la probabilidad de que las dos pelotas escogidas tengan
distinto color y compararla con la probabilidad de que tengan el mismo color.
Solucion. Para definir el espacio muestral conviene numerar las pelotas y pensar que las
rojas son la 1 y la 2, y que de la 3 a la 5 son azules; entonces el espacio muestral es

= {{1, 2}, {1, 3}, {1, 4}, {1, 5}, {2, 3}, {2, 4}, {2, 5}, {3, 4}, {3, 5}, {4, 5}}

y S = {{1, 3}, {1, 4}, {1, 5}, {2, 3}, {2, 4}, {2, 5}},
6
as que P (S) = 10 = 53 = 0.6. La probabilidad de que las dos pelotas tengan el mismo color
4
se calcula considerando el suceso T = {{1, 2}, {3, 4}, {3, 5}, {4, 5}}, y aqu P (T ) = 10 =
0.4 < P (S) (o, de otra manera, como T = S, P (T ) = 1 P (S) = 1 53 = 25 = 0.4).

7.10 Ejemplo. Como en el ejemplo anterior, se tiene una caja en la que hay 2 pelotas
rojas y 3 azules, pero ahora el experimento consiste en sacar una pelota, observar su color,
volverla a meter, y sacar otra vez una pelota. Calcular la probabilidad de que las dos pelotas
escogidas tengan distinto color.
Solucion. Numeremos las pelotas como en el ejemplo anterior. En este caso

= [5] [5] y

S = {(1, 3), (3, 1), (1, 4), (4, 1), (1, 5), (5, 1), (2, 3), (3, 2), (2, 4), (4, 2), (2, 5), (5, 2)},
por lo que P (S) = 12
25
= 0.48, que es menor que el resultado del ejemplo anterior, lo cual
resultaba intuitivamente obvio.

7.11 Ejemplo. Dentro de cierto grupo de 4 caballos numerados del #1 al #4 se ha


observado que la frecuencia con que el caballo #1 gana es el doble que con la que gana el
#2; que este a su vez gana el doble de veces que el #3, y que el #3 gana el doble de veces

36
que el #4. Encontrar la probabilidad de que en la proxima carrera el caballo ganador sea el
#3.
Solucion. Tenemos que representar en el espacio muestral las condiciones de que unos
ganan el doble de veces que otros. Podemos entonces asignar al caballo 4 el numero 1, al
caballo 3 los numeros 2 y 3, al caballo 2 los numeros 4, 5, 6 y 7, y al caballo 1 los numeros
2
del 8 al 15. De esta manera = [15], S = {2, 3} y la probabilidad es 15 0.13.

Para eliminar complicaciones tecnicas, en los dos ejemplos siguientes consideraremos


el ano con 365 das (sin contar en ningun caso el 29 de febrero) y supondremos que la
distribucion de los cumpleanos es pareja a lo largo del ano.

7.12 Ejemplo. Encontrar la probabilidad de que una persona determinada haya nacido
en enero o febrero.
59
Solucion. = [365], S = [59] y P (S) = 365 16 .

7.13 Ejemplo. Encontrar la probabilidad de que en un grupo de 59 personas al menos


2 tengan el mismo cumpleanos.
Solucion. Notemos que este ejemplo difiere del anterior en que las fechas de cumpleanos
no se comparan con fechas fijas sino entre s. Veremos que los resultados son muy distintos.
Para resolver el ejemplo resulta mas facil contar la probabilidad opuesta: que no haya ningun
cumpleanos repetido. Utilizaremos repetidamente la propiedad (3). Consideremos un orden
fijo para las personas. La probabilidad de que el segundo cumpleanos sea distinto del primero
es 364
365
. La probabilidad de que el tercero sea distinto de los dos anteriores es 363 365
, y as
sucesivamente. El resultado es
364 363 307
1 ,
36558
que es aproximadamente igual a 0.995. Esto quiere decir que de 1000 grupos de 59 personas
cada uno, se espera que en solo 5 de los grupos no haya cumpleanos comunes. (Comparese
este resultado con el del ejemplo anterior. Resulta que basta con 23 personas para que la
probabilidad de que haya cumpleanos repetidos entre ellas sea mayor que 21 .)

7.14 Ejemplo. Encontrar la probabilidad de que al lanzar una moneda al aire 10 veces
caigan exactamente 5 aguilas.
Solucion. Como antes, escribamos a por aguila y s por sol. El espacio muestral consta
de todas las sucesiones de longitud 10 formadas por a y s, de manera que || = 210 = 1024.
El suceso consta de los elementos de que tienen exactamente 5 a0 s, as que |S| es el numero
de formas en que se pueden escoger 5 posiciones (donde aparezcan las a0 s) dentro de un total
de 10, es decir, 10
5
252
= 252. Entonces P (S) = 1024 0.25.

En forma analoga a la resolucion del ejemplo anterior tenemos que la probabilidad de


que de un total de 20 lanzamientos de la moneda 10 salgan aguila es 2120 20

10
, que es apro-
ximadamente igual a 0.176. Se puede demostrar que mientras mas lanzamientos se hagan,

37
la probabilidad de que la mitad de las veces salga aguila es menor. Esto no contradice la
afirmacion de que si una moneda se lanza al aire un numero grande de veces se espera que
un numero cercano a la mitad de las ocasiones caiga aguila; la explicacion para esto es que la
idea de cercana debe manejarse en forma relativa al tamano del numero; por ejemplo, en
el caso de 10 lanzamientos podramos decir que los casos en que salieran entre 3 y 7 aguilas
son todos cercanos a la mitad, y en el caso de 20 lanzamientos diramos que los casos
cercanos a la mitad son entre 5 y 15.

7.15 Ejercicio. Encontrar la probabilidad de que al lanzar una moneda al aire 10 veces
salga aguila entre 3 y 7 veces.

Como ya hemos visto, se pueden considerar distintos espacios muestrales para resolver un
determinado problema y, en cada caso, el suceso del cual se quiere calcular la probabilidad
es diferente, as que los calculos tambien lo son, aunque, claro, el resultado final debe ser el
mismo. En el siguiente ejemplo presentamos varias formas de resolver el problema segun el
espacio muestral que se escoja.

7.16 Ejemplo. Cual es la probabilidad de que al escoger dos subconjuntos de 4


elementos dentro de un conjunto de 10 elementos, los subconjuntos tengan al menos un
elemento en comun?
Solucion. Es mas facil contar la probabilidad contraria, es decir, la probabilidad de que los
dos subconjuntos escogidos no tengan elementos en comun. Consideremos distintos espacios
muestrales y los respectivos sucesos S con complemento S:

Primera forma. Sea P4 = {A [10] : |A| = 4}, es decir, P4 tiene por elementos a
los subconjuntos de [10] que tienen 4 elementos. Tomemos = P4 P4 . En este caso
10 6
 
|S| = 4 4 , as que
6543
10 6 6
  
4 4 4
4321
109874321 6543 13
P (S) = 1 P (S) = 1 10 2
=1 10
 =1 1 = .
= 10 9 8 7 14

4 4

Segunda forma. Sea P4 como arriba. Supongamos que un conjunto de 4 elementos ya esta
escogido; entonces queremos calcular la probabilidad de que al escoger otro conjunto, este
sea ajeno con el primero. En este caso tomemos = P4 . Aqu |S| = 64 y |M | = 10 4
, as
que
6543
6

4
4321
109874321 6543 13
P (S) = 1 P (S) = 1 10 = 1 1 = .
4
= 10 9 8 7 14
Tercera forma. Como en la segunda forma, supongamos que un conjunto de 4 elementos
ya esta escogido; entonces queremos calcular la probabilidad de que al escoger otro conjunto,
este sea ajeno con el primero. Sea = {(c1 , c2 , c3 , c4 ) [10] : ci 6= cj para i 6= j}. y
supongamos que el conjunto ya escogido tiene elementos a1 , a2 , a3 , a4 ; en este caso S =

38
{(c1 , c2 , c3 , c4 ) : para cada i, j ci 6= aj }, |S| = 6 5 4 3 y |M | = 10 9 8 7 y, entonces,
6543 13
P (S) = 1 P (S) = 1 = .
10 9 8 7 14

7.17 Ejemplo. Alejandra y Delia van a jugar un juego. Alejandra lanzara un dado y
le dara una moneda a Delia cada vez que lo que salga en el dado no sea 2. Si se quiere que
ninguna de las dos jugadoras tenga ventaja sobre la otra, cuantas monedas debera pagar
Delia cada vez que salga el 2?
Solucion. Como la probabilidad de que salga el 2 es 61 , se espera que de cada 6 veces una
de ellas salga 2; entonces Delia debera darle 5 monedas cuando esto ocurra. En 6 juegos se
espera que Alejandra pierda 5 veces una moneda y gane una vez 5 monedas, por lo que su
ganancia esperada es de 0.

Generalicemos el ejemplo que acabamos de estudiar. Si algo puede ocurrir de un total


de r formas (mutuamente excluyentes) con probabilidades p1 , p2 , . . ., pr (de manera que
p1 + p2 + + pr = 1) y ganancias respectivas g1 , g2 , . . ., gr , entonces el valor esperado E
del suceso se define como
E = g1 p1 + g2 p2 + + gr pr .
Para entender mejor esta nueva definicion analicemos en el ejemplo anterior cual es la ga-
nancia esperada de Alejandra si Delia le da 5 monedas cada vez que salga el 2. Llamemos p1
a la probabilidad de que no salga el 2 y p2 a la probabilidad de que s salga; entonces p1 = 65 ,
p2 = 16 , g1 = 1 (pues Alejandra pierde una moneda cuando no sale el 2) y g2 = 5. El valor
esperado del suceso (ganancia esperada para Alejandra) es E = (1) 56 + 5 61 = 0.

7.18 Ejemplo. En el juego de ruleta hay 36 numeros (del 1 al 36) y ademas los smbolos
0 y 00, con los que el dueno de la ruleta gana automaticamente. Se ofrece pagar 36 veces lo
apostado cada vez que salga el numero al que uno aposto (es decir, si uno indica uno de los
36 numeros y paga una ficha por jugar, en caso que al girar la ruleta la bolita se detenga en
el numero escogido, el dueno de la ruleta le devolvera su ficha al jugador y le dara otras 35
mas). Cual es la ganancia esperada de un jugador?
Solucion. Llamemos p1 a la probabilidad de que el jugador gane, y p2 a la probabilidad
1
de que pierda. Entonces p1 = 38 , p2 = 37
38
1
, g1 = 35 y g2 = 1; as E = 35 38 37
+ (1) 38 =
2
38
, que es aproximadamente igual a 0.05. Esto quiere decir que el jugador espera perder
alrededor de un 5 % de lo apostado; en otras palabras, el dueno de la ruleta espera ganar el
5 % de lo que se apueste.

7.19 Ejercicio. () Cuatro equipos A, B, C, D entran a un torneo de basquetbol. Al


principio juegan A contra B, y C contra D; en cada juego se elimina al perdedor. Los dos
ganadores se enfrentan y el que gane ese juego se determina como ganador del torneo. Escribir
un espacio muestral apropiado y el suceso correspondiente para determinar la probabilidad
de que B sea el ganador.

39
7.20 Ejercicio. () Un grupo de 3 mujeres y 3 hombres se dividira en dos equipos con
3 miembros cada uno. Definir un espacio muestral y el suceso correspondiente que sirvan
para encontrar la probabilidad de que en uno de los equipos queden todos los hombres y en
el otro todas las mujeres.

7.21 Ejercicio. () Se dice que una mano de domino tiene falla si alguno de los
numeros entre el 0 y el 6 no aparece en la mano (cada numero faltante es una falla); por
ejemplo la mano {2|1, 5|5, 3|1, 0|0, 1|0, 5|6, 0|2} tiene falla a 40 s. Cual es la probabilidad de
que una mano de domino no tenga falla?

7.22 Ejercicio. () Una persona quiere apostar que la suma de lo que muestren dos
dados es cierto numero. A que numero le conviene apostar?

7.23 Ejercicio. () Se eligen al azar n cartas de la baraja. Como debe ser n para que
la probabilidad de que entre las cartas elegidas haya (al menos) dos del mismo numero sea
mayor que 21 ? Cual es la probabilidad si n = 14?

7.24 Ejercicio. () Calcular la probabilidad de que al lanzar tres veces dos dados, las
tres veces los numeros que salgan sean iguales entre s.

7.25 Ejercicio. () Se escogen al azar en sucesion tres numeros (posiblemente iguales)


entre el 1 y el 100. Cual es la probabilidad de que se hayan escogido en orden creciente
estricto?

7.26 Ejercicio. () Un dado se lanza al aire 6 veces. Cual es la probabilidad de que


aparezca cada uno de los seis numeros una vez?

7.27 Ejercicio. () Supongamos que de un grupo de 10 enfermedades cada una tiene


1
probabilidad 10 de atacar a un animal determinado a lo largo de su vida. Que probabilidad
tiene ese animal de enfermarse de al menos una de esas enfermedades?

7.28 Ejercicio. En cierto examen de opcion multiple con 5 opciones en cada respuesta
se califica como sigue: por cada respuesta correcta se otorga +1 punto, por cada respuesta
incorrecta se otorga 14 de punto, y por cada pregunta sin contestar se otorgan 0 puntos.
Que calificacion esperara obtener alguien que contestara todo el examen?

40
8. Principio de las Casillas

A primera vista el Principio de las Casillas (tambien llamado Principio de los Palo-
mares) parece extremadamente simple e inofensivo. Sin embargo tiene muchas aplicaciones
en problemas de del tipo que nos interesan en estas notas.

8.1. Principio de las Casillas. Si se dispone de n casillas para colocar m objetos y


m > n, entonces en alguna casilla deberan colocarse por lo menos dos objetos.

Empezaremos ilustrando con un ejemplo muy simple y tpico de aplicacion varias veces
del Principio de las Casillas.

8.2 Ejemplo. Un costal esta lleno de canicas de 20 colores distintos. Al azar se van
sacando canicas del costal. Cual es el mnimo numero de canicas que deben sacarse para
poder garantizar que en la coleccion tomada habra al menos 100 canicas del mismo color?
Solucion. Notemos que si sacaramos 20 canicas, podra ser que todas fueran de colores
distintos, as que solo podramos garantizar que hay dos canicas del mismo color si sacaramos
21 canicas (aqu se aplico el Principio de las Casillas). De la misma manera, necesitaramos
41(= 20 2 + 1) canicas para poder afirmar que con seguridad hay 3 canicas (al menos)
del mismo color, pues con 40 canicas podra ser que cada color apareciera exactamente 2
veces. Con el mismo razonamiento que hemos seguido llegamos al resultado: Se necesitan
20 99 + 1 = 1981 canicas.

8.3 Ejercicio. En un papel cuadriculado de 69 cuadrados se consideran 25 triangulos


arbitrarios y diferentes que tienen sus vertices en los puntos de interseccion de las lneas de
la cuadrcula. Mostrar que no importa como se elijan los triangulos, forzosamente habra (al
menos) dos triangulos con (al menos) un vertice en comun.

Los siguientes ejemplos son todos de naturaleza muy distinta; sin embargo, en la solucion
de todos ellos utilizamos el Principio de las Casillas.

8.4 Ejemplo. Algunos de los cuadritos de una cuadrcula de 3 7 se pintan de negro


y los otros se dejan en blanco. Probar que forzosamente las lneas de la cuadrcula forman
un rectangulo en cuyas cuatro esquinas los cuadraditos tienen el mismo color (los cuatro
blancos o los cuatro negros).
Solucion. Supongamos que tenemos una cuadrcula pintada de manera tal que no se
forma el rectangulo con las esquinas del mismo color. Simbolicemos por N al color negro
y por B al color blanco, y observemos que los cuadritos de una columna pueden haber
quedado pintados segun las siguientes 8 posibilidades: p1 = N N N , p2 = N N B, p3 = N BN ,
p4 = BN N , p5 = N BB, p6 = BN B, p7 = BBN y p8 = BBB. Supongamos que una de las
columnas esta pintada segun la posibilidad p1 ; entonces con cualquiera de las posibilidades
en que la columna tiene dos N 0 s se formara un rectangulo con las esquinas negras, as que

41
ninguna columna esta pintada as; pero entonces las columnas estan solo pintadas segun las
posibilidades p1 , p5 , p6 , p7 y p8 ; como el numero de columnas es 7, entonces el Principio de las
Casillas nos dice que debe haber dos columnas iguales, pero aqu tambien, por el Principio
de las Casillas, como son tres cuadritos en cada columna y solo dos colores, hay un color
que se repite, y entonces es obvio que se forma un rectangulo con las esquinas del mismo
color. Concluimos entonces que la posibilidad p1 no aparece. Lo mismo ocurre al considerar
la posibilidad p8 . Entonces ninguna de las posibilidades p1 y p8 aparece; pero as sobran
solo 6 posibilidades, con lo cual, otra vez aplicando el Principio de las Casillas, tenemos dos
columnas iguales, y de ah una contradiccion.

8.5 Ejercicio. Algunos de los cuadritos de una cuadrcula de 19 4 se pintan de rojo,


otros de azul y otros de verde (no se deja ninguno en blanco). Probar que forzosamente las
lneas de la cuadrcula forman un rectangulo cuyas cuatro esquinas tienen el mismo color.

8.6 Ejemplo. Probar que en cualquier conjunto de 6 personas forzosamente hay 3 que
se conocen todas entre s o 3 tales que ninguna conoce a las otras 2.
Solucion. Por cada una de las personas pongamos un punto e indiquemos que dos personas
se conocen poniendo una lnea entre los puntos que las representan, obteniendo as la grafica
de conocidos. Si A es una de las personas, las restantes 5 se pueden dividir en dos grupos:
las conocidas de A y las desconocidas de A. Por el Principio de las Casillas, alguno de los
grupos tendra 3 o mas elementos. Primero supongamos que el de los conocidos de A tiene
tres o mas elementos y sean B1 , B2 y B3 conocidos de A.

A
........
... ..........
... .... ............
... ... .......
..... ... .......
.......
... ... .......
... ...

.
.
.
..
.
.
..
.
.
.
.
.
...
. B2
...... .... ....
.......
.......
.......
.......
.
... .... ... .... ....
.... .... ............
...
. ..
. .... .... .... .......
.
... .
.... ............
..
... .. ...
. .... .... .... ....
... .. .... .. .... .... .... ....
..
.... .
..
........... .... .... .... .... .... .. .. ...
. .... ....
B3
B1

Si dos de B1 , B2 y B3 se conocen entre s, entonces A junto con esos dos formara el grupo
de los tres conocidos que buscabamos. Si no, entonces B1 , B2 y B3 formaran el grupo de los
tres desconocidos que queramos. El caso en que el numero de desconocidos de A sea 3 o mas
se trata de manera analoga al anterior, considerando tres desconocidos C1 , C2 y C3 de A.

El ultimo ejemplo de esta seccion nos servira, ademas de para ver una aplicacion intere-
sante (y muy simple) del Principio de las Casillas, para recordarnos algunas propiedades
importantes de los numeros. Recordemos que los numeros reales son aquellos que nos sirven
para medir con direccion, es decir, aquellos que se pueden representar en una recta una
vez que se ha establecido un punto de partida (el 0) y se ha indicado otro punto distinto (el
1) que nos senala precisamente la medida 1 y que la direccion del 0 al 1 es la positiva; as,

42
los numeros reales del mismo lado del 0 que el 1 son los positivos, mientras que los reales
del lado opuesto son los negativos. Entonces tenemos que todo
numero entero es real, pero
hay reales que no son enteros, como por ejemplo 1.5, 2.47, 2 y .
negativos
....................................................
positivos
.......................................................
..

| | | | | |
........................................................................................................................................................................................................................................................................................................................................................................................................................................

3 1.5 0 1 2 2.47

El conjunto R de los numeros reales se descompone en dos conjuntos ajenos (es decir,
dos conjuntos sin elementos en comun): el de los numeros racionales (denotado por Q) y el
de los numeros irracionales (denotado por I). Los numeros racionales son los que se pueden
expresar como cociente de enteros, y los irracionales son los que no. Por ejemplo, son numeros
racionales 1.5, 0, 3 y 2.47, pues se pueden expresar, respectivamente, como 3 , 0 , 3 y 100
247
.
2 1 1
Utilizando resultados de divisibilidad de enteros no es difcil probar que 2 no es racional.
Un resultado conocido pero difcil de demostrar es que tampoco es racional. La escritura
conocida con cifras de un numero real cualquiera se llama expansion decimal del numero
(o sea, la expansion decimal del numero 23 053.04 es, precisamente, 23 053.04). La razon
para esta nomenclatura es que la expresion se basa en un sistema posicional en que en cada
posicion la cifra (que puede ser cualquier entero entre 0 y 9) representa el numero de veces
que debe tomarse la potencia de 10 correspondiente a esa posicion; por ejemplo:
1
23 053.04 = 2 104 + 3 103 + 5 101 + 3 100 + 4 2 .
10
No debemos pasar por alto que algunas expansiones decimales son infinitas (despues del
punto decimal), como por ejemplo la expansion de 13 , que consta de una infinidad de 30 s.
Esto usualmente se representa por 13 = .3, indicando que el 3 debe repetirse una infinidad de
veces. De esta manera, tambien podemos considerar numeros reales como 1.2403 en donde,
a partir del 2 la repeticion es de las tres cifras 403. En los casos que acabamos de considerar
en que hay una repeticion infinita de cifras, decimos que la expansion de los numeros es
periodica y llamamos periodo a lo que se repite (en el primer caso el periodo es 3 y en
el segundo, el periodo es 403). Podemos decir que las expansiones finitas son periodicas con
periodo 0 (por ejemplo, 2.4 = 2.40). Es importante notar que hay numeros reales que no
tienen expansion periodica, como por ejemplo el numero .1010010001..., en el que despues
del punto los 10 s estan intercalados entre cada vez mas 00 s. No es difcil probar que: Si un
numero real tiene una expansion periodica, entonces el numero es racional: Si el numero se
llama x, lo podemos multiplicar por potencias apropiadas de 10, de tal manera que al restar
una de otra se elimine el periodo, y despues despejar x de una expresion de enteros (por
ejemplo, si x = 3.825, entonces 103 x 10x = 3825.25 38.25 = 3787, as que x = 3787 990
). El
contenido del siguiente ejemplo, que demostraremos utilizando el Principio de las Casillas,
es, precisamente, el recproco de este resultado.

8.7 Ejemplo. Probar que la expansion decimal de cualquier numero racional es pe-
riodica.

43
Solucion. Sea ab el numero considerado, donde a y b son enteros y b 6= 0. Al hacer la
division segun el algoritmo usual, los residuos que van quedando son enteros entre 0 y b 1,
as que forzosamente debera haber alguna repeticion; a partir de ese momento, los cocientes
y los residuos que se van obteniendo van formando un periodo de repeticion.

8.8 Ejercicio. Sea A un conjunto de 19 enteros (distintos) escogidos dentro de las


sucesion aritmetica 1, 4, 7, . . . , 100. Probar que en A hay dos elementos (distintos) cuya suma
es 104.

8.9 Ejercicio. La suma de 5 enteros positivos es 100. Probar que al menos hay dos de
esos numeros cuya diferencia es menor o igual que 9.

8.10 Ejemplo. Sea p 6= 2, 5 un numero primo. Probar que hay una infinidad de multi-
plos de p en la sucesion (1, 11, 111, 1111, . . .).
Solucion. Sea ar el numero formado por una cantidad r de 10 s. Gracias al Principio de las
Casillas, en la lista de residuos modulo p de la lista infinita 1, 11, 111, 1111, . . . debe haber
repeticion, es decir existen r < s naturales tales que ar as (mod p), de donde as ar es
multiplo de p. Pero as ar = 10r asr y p es primo relativo con 10, as que asr es multiplo
de p. Hasta aqu hemos encontrado un multiplo de p dentro de la lista. Para encontrar una
infinidad observemos que el numero formado por una cantidad multiplo de s r de 10 s
tambien es multiplo de p (para ver esto basta aplicar el conocido algoritmo para hacer una
division que aprendimos desde ninos).

8.11 Ejemplo. Sean a1 , a2 , . . . , a10 enteros. Probar que existen

1 , 2 , . . . , 10 {1, 0, 1}

no todos cero tales que 1 a1 + 2 a2 + + 10 a10 es multiplo de 1000.


Solucion. Consideremos {1 a1 + 2 a2 + + 10 a10 : i {0, 1}}. Las posibilidades para
expresar estos elementos son 210 = 1024; como hay solo 1000 residuos posibles modulo 1000,
entonces, por el Principio de las Casillas, tenemos que dos de estos son congruentes modulo
1000, de donde su diferencia es multiplo de 1000. Ahora solo observemos que la diferencia
es una expresion como la que estamos buscando.

8.12 Ejercicio. () En un torneo de futbol cada equipo juega una vez exactamente
con cada uno de los demas. Los juegos estan se realizan siempre los domingos. Probar que
cualquier lunes siempre hay dos equipos que han completado exactamente el mismo numero
de juegos.

8.13 Ejercicio. () Probar que si cada punto del plano se colorea de rojo o azul,
forzosamente habra un segmento de longitud 1 cuyos extremos tengan el mismo color.

44
8.14 Ejercicio. () En un congreso internacional se reunen n cientficos de 6 pases.
Durante el congreso los cientficos se dividen en cuatro secciones de tal manera que dentro
de cualquier grupo de 6 participantes de la misma seccion siempre hay dos cientficos de la
misma edad. Encuentra el mnimo numero n para el cual, bajo las condiciones mencionadas
arriba, se puede asegurar que existen tres cientficos de una misma seccion que tienen la
misma edad y pertenecen al mismo pas.

8.15 Ejercicio. () Sea M un conjunto de 9 enteros ninguno de los cuales tiene un


divisor primo mayor que 6. Probar que hay dos elementos de M cuyo producto es un cua-
drado.

8.16 Ejercicio. () Isabel escoge 8 puntos de los marcados. Cual es la probabilidad


de que cuatro de los puntos escogidos sean los vertices de un rectangulo?

45
9. Coloracion

En varios de los problemas de esta seccion se estudia la posibilidad de cubrir tableros de


un determinado tamano o forma con fichas que cumplen ciertas caractersticas. Se entiende
que las fichas no deben traslaparse ni cubrir area fuera del tablero. Se pide ademas que
entre todas las fichas se tape todo el tablero. Para ilustrar, consideremos el caso de cubrir un
tablero de 88 con fichas de tamano 21. Es muy facil lograrlo y en la figura esquematizamos
2 formas de hacerlo.
.................. .................. .................. .................. .................. .................. ..................
.... ....
... ...
... ...
.................. .................. .................. .................. ... .................. ... .................. . .
... ...
.. ..
.................. .................. .................. .................. ... ... ..................
..... ..... ..... .....
... ... ... ...
... ... .. ...
.................. .................. .................. .................. .. . . .. .. .. . .
... ... ... ...
.. .. .. ..
.................. .................. .................. .................. ... .................. ... ... ... ..................

.................. .................. .................. .................. .................. .................. .................. ..................

.................. .................. .................. .................. .................. .... .................. .................. ....


... ...
.. ..
.................. .................. .................. .................. .................. .. .................. .................. ..

Desde luego, el ejemplo anterior fue muy sencillo y no tuvo ningun interes teorico. Sin
embargo una ligera variacion lo vuelve mucho mas interesante, como veremos a continuacion.

9.1 Ejemplo. Utilizando fichas de tamano 2 1, es posible cubrir un tablero de 8 8


del que se han quitado dos esquinas opuestas (de 1 1)?
Solucion. Podramos hacer muchos intentos sin lograr cubrir el tablero; sin embargo esto
no nos demostrara que no es posible. Utilizamos un argumento ingenioso: pintar el tablero
como si fuera un tablero de ajedrez (alternando los colores blanco y negro). Observamos
entonces que a las dos esquinas que se eliminaron les corresponda el mismo color, as que
quedaron mas cuadros de un color que de otro. Por otro lado, sin importar el lugar donde se
coloquen las fichas de 2 1, cada una de ellas cubre un cuadro negro y uno blanco as que,
si se pudiera cubrir el tablero, el numero de cuadros blancos cubierto sera el mismo que el
de negros. Este argumento demuestra que no es posible cubrir el tablero.
...............................................................................................................................................................
... ....................................... ...................................... ...................................... ...
... ....................... ....................... ........................ ...
................................................................................... ........................................ ..........
.......................... ............................ .................... ....................
.......................... ............................ .......................... ..........................
................... ..................... .......................... ..........................
........................................................................................................................
................................... .......................... ............................ ....................
............................ . .................. . .................... ..........................
............................ ............................................................. ...... .......................... ......
................................. ............................ .................... ....................
.......................... ............................ .......................... ..........................
.......................... ............................ .......................... ..........................
. . . . . . . . . . . . . . ............ ......
................................... ......................... ........................... ..........................
.
.
......................... .
........................... . . . .
................... . .
............................ . . . . . . . . .
............................ .........................
. ...........................
.
...................
. . . . . . .
....................................................................................................
...... ....... ................................. .................................
................................. ...................................
.......................... ..................... .......................... ..........................
................................................................................................................
................................... . .................. . .................... ....................
............................ ...................
. ......
....................
. ....... . .................
.
..................... .................... ..................... ..........................
......................................................................................................................................
... ....................................... ....................................... .................................... ...
... ................................ ................................ .............................. ..
...........................................................................................................................................................

46
9.2 Ejemplo. A un tablero cuadriculado de 99 se le han quitado tres de sus esquinas
(de 1 1). Es posible cubrirlo con fichas de 3 1?
Solucion. Utilizamos ahora 3 colores que indicamos en la figura con los numeros 1, 2 y 3.
........................................................................................................................................
.... .... .... .... .... .... .... ...
....
.. 2 ....
.. 3 ....
.. 1 ....
.. 2 ....
.. 3 ....
.. 1
....
..
.... 2
....................
...
3 1 2 3 1 2 3 1
.....................
...
.. 2
...
2 3 1 2 3 1 2 3
......................
...
. 1
...
1 2 3 1 2 3 1 2
.....................
3
...
.
...
3 1 2 3 1 2 3 1
.....................
2 ...
.
...
2 3 1 2 3 1 2 3
.....................
1 ...
..
...
1 2 3 1 2 3 1 2
.....................
3 ...
.
...

.
3 .
1 .
2 .
3 . .
1 .
2 .
3 1
....................
2 ...
..
.... .... .... .... .... .... .... .... ...
...
.2 3
...
.
...
. 1 ...
. 2 ...
. 3...
.
...
.1 ...
. 2
..................................................................................................................................................................
....
. 3

En este caso, cualquier colocacion de las fichas de 13 cubre una vez cada uno de los colores,
as que en la coloracion debera haber la misma cantidad de cuadros de cada color; pero esto
no es cierto ya que en la cuadrcula de 9 9 s haba la misma cantidad de cuadros de cada
color, pero a dos de los tres cuadros que se quitaron les corresponda el mismo color (el 1).
(notemos que hay que escoger la coloracion en forma adecuada; por ejemplo, si se hubiera
quitado la esquina inferior izquierda en lugar de la derecha, la coloracion dada no servira
pues se quitara un cuadro de cada color, en ese caso hubiera convenido comenzar el segundo
renglon con el color 2 y despues seguir alternando de la misma manera.)

9.3 Ejercicio. Rehacer el ejemplo anterior utilizando solo dos colores.

9.4 Ejercicio. () Es posible construir un rectangulo con las cinco fichas que se
muestran a continuacion (todos los cuadritos marcados son de 1 1)?

.................... ......................
...
.... .....
.. . .....
.................... ................... .................... ... .
.. ...
..
. ....
.................... ...................

9.5 Ejercicio. () La estrella que se muestra abajo consta de 108 triangulos equilateros
iguales. Es posible cubrirla con fichas de 4 triangulos equilateros (del mismo tamano que
los de la estrella) como la que se muestra al lado de la estrella?

47
..
......
... ...
... ...
..............................
.. .......
... ...
... .... ..... .....
... . . .
..........................................................
.
... ... ...... .. ..
. ... .... ..... ..... ..... .....
... . ...
... . ... ...
.............................................................................................................................................................................................................................................
... .. . . . .. . . .
... .... ..... ..... ..... ..... ..... ..... ..... .... ..... ..... ..... ..... ..... ..... ..... .....
... ... ... .. ... .. ... ... ... .. ... .. . . . . . .
.......................................................................................................................................................................................................................
... .... ....
. . ... ..... .... .... .... .
... ... . .
. . .. . ... . . . ..
. . .
.. . ...
... .... ..... ..... ..... ..... ..... .... ..... ..... ..... ..... ..... ..... ..... .....
... ... . . . . . . . . . . . . . .
..........................................................................................................................................................................................................
... .. . .. .. .. ..
... ..... ..... ..... ..... .... ..... ..... ..... ..... ..... ..... ..... .....
... .. ... ... ... ... ... .. ... .. ... .. ... .. ............................................................
... ... . . . . . . . . . . . .
........................................................................................................................................................................... ... ... . . .
.
. . . . . . .. ..... ... ..... ...
... ...
.
... ... ... ...
.
... ...
.
.. ...
. .
.. ... ... ... .. .. ..
... ..... .... ..... .... .... .... ..... ..... .... ..... ..... .... ..... ....................................................
..
... ...... ...... ... .. ..... ... .. ...... ..
.................................................................................................................................................................................
. .. . . . . .. . . . .
...... .. . . . . . . . .. . .
... ... ... ... .. ... ... ... ... ... ... ... ... ... .. ...
.... .... ..... ..... ..... ..... ..... .... .... ..... .... ..... ..... ..... ..... .....
.
..
. .........................................................................................................................................................................................................
.. ... .. .. .... ... .. .. ....
... .... ... ... ... .... ... .... ... ... ... ... ... .... ... .... ... ...
... ..... ..... ..... ..... ..... ..... ..... .... ..... ..... ..... ..... ..... ..... ..... .... .....
... .. .. .. .. .. .. .. .. .
...........................................................................................................................................................................................................................................
... .. .. .
... .... ..... .... ..... .....
... ... ... .... . .
.
..........................................................
...
... ....... ...
... ..... ..... .....
... .. . .
................................
... .
... .....
... ...
......
..

9.6 Ejercicio. () Se considera el conjunto S de puntos del espacio formado por los
vertices de un cubo y los puntos medios de las caras del cubo. Se ponen segmentos entre los
puntos de S uniendo cada punto medio de una cara con los 4 vertices del cubo que forman
esa cara. Es posible hacer un recorrido sobre los segmentos dibujados que pase exactamente
una vez por cada uno de los puntos de S?

9.7 Ejemplo. Segun un diseno ya establecido, un piso rectangular se va a cubrir con


algunos mosaicos de 22 y otros de 14. Se rompio un mosaico y el distribuidor lo sustituyo
por otro del tipo contrario. Probar que no se puede redisenar el arreglo.
Solucion. Cuadricular el piso, y en los renglones pares colorear alternadamente con negro
y blanco. Dejar los renglones impares en blanco (ver figura).
.... .... .... .

.... .... .... .


...........................................................................
. .
............................................................................ ................................................................... ....................................................................
............................................................. .............................................................. ................................................................ ................................................................
.......................................................... .............................................................. ................................................................ ................................................................
.................................................................. ............................................................... .................................................................. ...................................................................... .... .... .

......................................... ............................................. .................................................. .......................................................... .... .... .


................................................................ ................................................................... .................................................................. ................................................................
.................................................................. ................................................................ .................................................................. .................................................................
.................................................................. .................................................................. ................................................................ ...............................................................
............................................ ............................................. .................................... ............................................ .... .... .

... ... ... ... ... ... ... ...... .... .... .
... ... ... ... ... ... ... ...
... ... ... ... ... ... ... ...
. . . . . . . .

Entonces, segun el primer diseno, cada mosaico de 2 2 cubrira exactamente un cuadro


negro, mientras que los de 14 cubriran 0 o 2 y as la paridad de los cuadros negros cubiertos
sera distinta al cambiar un mosaico.

Para ver que s es posible lograr exactamente 9 cuadros vacos, en el siguiente dibujo
hemos indicado con una lnea diagonal el que las fichas que estan en las casillas que une la
diagonal se intercambian, dejando entonces vacas las casillas indicadas con .

48
9.8 Ejercicio. () Probar que si hay n puntos en el plano, con n 4, entonces es
posible colorearlos con dos colores de manera que no exista ninguna recta que deje todos los
puntos de un color de un lado y los del otro color del otro lado (y que no toque a ninguno
de los puntos).

Los siguientes ejemplos tratan coloraciones de cantidades infinitas de puntos.

9.9 Ejemplo. Probar que si cada punto del plano se colorea de rojo o azul, forzosa-
mente habra un segmento de longitud 1 cuyos extremos tengan el mismo color.
Solucion. Primera forma. Tomemos un triangulo equilatero de lado 1. Dos de sus vertices
deben ser del mismo color.
Segunda forma. Tomemos un punto, digamos, rojo y consideremos el crculo con centro
en ese punto y radio 1. Si en el crculo hay un punto rojo, ya acabamos; si no, entonces todo
el crculo es azul y en el es claro que hay una cuerda de longitud 1.

9.10 Ejemplo. Probar que si cada punto del plano se colorea de azul, rojo o verde,
entonces uno de los tres colores tiene una pareja de puntos a distancia 1.
Solucion. Supongamos que no. Entonces todo triangulo equilatero de lado 1 tiene sus
tres vertices de distinto color. Tomemos uno de estos triangulos y sea O un vertice rojo.
Del otro lado de la base que tiene por extremos un punto verde y otro azul pongamos otro
triangulo; entonces el nuevo vertice debe ser tambien rojo. Esto es cierto para cualquier
triangulo equilatero uno de cuyos vertices sea O, es decir, el crculo con centro O
y radio 1
tiene tiene todos sus puntos azules o verdes y el crculo con centro O y radio 3 es todo
rojo. Es claro que este crculo tiene una cuerda de longitud 1 y as encontramos dos puntos
rojos a distancia 1.

9.11 Ejercicio. () Cada punto del espacio es rojo, azul, blanco o verde. Probar que
hay un segmento unitario cuyos extremos son del mismo color.

49
10. Graficas

Una grafica (finita, simple) G (consta de un conjunto (finito) V = V(G) cuyos elemen-
tos se llaman vertices, y de otro conjunto A = A(G) que consta de subconjuntos de dos
elementos de V llamados aristas. Escribimos G = (V, A). Si a = {u, v} A, escribimos
a = uv y decimos que u y v son extremos de a.

Por el momento solo trabajaremos con este tipo de graficas y dejaremos para despues el
estudio de multigraficas en donde se permite que A sea multiconjunto (es decir, que entre
dos vertices haya mas de una arista), o de seudograficas en las que, ademas, se permite que
las aristas consten de un solo elemento de V (una arista as se llama lazo), o el de digraficas
(o multi/seudo digraficas) en las que las aristas son parejas ordenadas de vertices. En este
curso no trabajaremos con graficas infinitas (aquellas en las que el conjunto de vertices es
infinito).

En general se hace una representacion geometrica de la grafica poniendo un punto por


cada vertice y representando cada arista por una lnea entre los dos vertices que la determinan
(en el caso de digraficas se pone una flecha).

De aqu en adelante G denota una grafica cualquiera con conjunto de vertices V y conjunto
de aristas A. Los vertices de G seran u, v, w, . . ..

Si v es un vertice, el grado de v es el numero (v) de aristas que tienen como elemento


a v. Definimos (G) = max{(v) : v V} y (G) = min{(v) : v V}. La grafica G es
r-regular si (G) = r = (G).

10.1 Proposicion. La suma de los grados de los vertices de una grafica es el doble del
numero de aristas.
Demostracion. Al sumar los grados de los vertices, cada arista se cuenta dos veces (una
por cada uno de sus extremos).

10.2 Ejercicio. Probar que el numero de personas en el mundo que tienen un numero
impar de hermanos es par.

10.3 Ejercicio. () Sea G una grafica regular. Si G tiene 26 aristas, cuantos vertices
puede tener?

10.4 Ejercicio. () Se quiere disenar una competencia con n participantes en la que


cada uno compita exactamente con otros k. Probar que una competencia tal puede disenarse
si y solo si nk es par y n k + 1.

Si a = uv A, decimos que u y v son adyacentes y que a es incidente a u y a v. Si

50
dos aristas tienen exactamente un vertice en comun decimos que son adyacentes.

Dados u, v V, un camino C de u a v (o uv-camino) es una sucesion de vertices


alternados con aristas C = (u = v0 , a1 , v1 , a2 , . . . , an , vn = v) tal que para cada i = 1, . . . , n, ai
es incidente a (los vertices distintos) vi1 y vi . Tambien escribimos C = (u = v0 , v1 , . . . , vn =
v) o C = (a1 , . . . , an ). El camino es cerrado si v0 = vn ; si no, se dice que es abierto. El
camino es trayectoria si no repite vertices; es paseo si no repite aristas y es ciclo si no se
repiten aristas y los unicos vertices que se repiten son el primero y el ultimo (v0 = vn ). La
longitud del camino, l(C), es n (el numero de aristas).

10.5 Ejercicio. () Probar que todo uv-camino C contiene una uv-trayectoria. (Suge-
rencia: Proceder por induccion sobre la longitud de la trayectoria.)

Una grafica es conexa si dados cualesquiera dos vertices u y v existe un uv-camino (o


uv-trayectoria). En caso contrario se dice que la grafica es disconexa.

Dos graficas G1 = (V1 , A1 ) y G2 = (V2 , A2 ) son isomorfas si existe una biyeccion f :


V1 V2 tal que uv A1 si, y solo si, f (u)f (v) A2 . Cuando dos graficas son isomorfas,
las consideramos iguales; as por ejemplo podemos hablar de la grafica que consta de un solo
vertice. Es claro que dos graficas isomorfas comparten las mismas propiedades, por ejemplo
de conexidad o del grado de los vertices (mas aun, un isomorfismo debe mandar un vertice
a otro del mismo grado). Denotamos por Tn la trayectoria de longitud n, y por Cn al ciclo
de longitud n.

Decimos que una grafica H es subgrafica de G (o que esta contenida en G) si el


conjunto de vertices y de aristas de H son subconjuntos del conjunto de vertices y del de
aristas de G, respectivamente (o si hay un isomorfismo entre H y una subgrafica de G).
Una componente conexa de G es una subgrafica conexa maximal (es decir, ella es conexa,
pero cualquier otra subgrafica que la contenga propiamente es disconexa). La componente
conexa de un vertice v es la subgrafica conexa maximal que lo contiene.

10.6 Ejemplo. Hay 6 graficas conexas no isomorfas con 4 vertices.


Demostracion. Son las siguientes 6:

......
...............................
.....
... ...
...................................... .................................................... ................................................................
... .....
............................................................................. ..
...
. ...
..
. ...
...
...
...
...
..
....
.
...
... .........
.. ......
.. ...
...
..
... ....... ..
... .............. ...
.. ...... ...... ...

......................................................
..............................
.............................
................................ ....................................

51
10.7 Ejercicio. () Dar un ejemplo de dos graficas conexas no isomorfas en las que
los grados de los vertices sean los mismos.

La grafica completa con n vertices es aquella en la que por cada par de vertices hay
una arista que los une (es decir, tiene n2 aristas); denotamos esta grafica por Kn . Si G es
una grafica con n vertices, la podemos considerar como subgrafica de Kn y el complemento
G de G es la subgrafica de Kn formada por los mismos vertices de G pero en el que las aristas
son aquellas aristas de Kn que no son aristas de G.

....... .....
................................................................................................ ...........
........ ........
...... .. ... .....
... .... ..... .... .... ...........
... ..... . . ...
.... .. ... .... ......... ..... ....
. ...... .. ..... ......
.
..... ..... .. ......
...
..
...
...
...
...
...
...
..
.....
..... ........
..... .....
...
... ... ....... . ... .... .
.................................................................................................
...
...
...
.
...
.
.
... ...
...
...
...
...
... .....
. ..
..
.
..
............
.....
.
...
...
...
... ........ ....
...
...
.......
... ........... ...
............. ...............
.
.......
..
...
...
.
..
...
... .........
.. .
.....
..... .... ... .. ... ... . ..
..
. ... ... ... .................... .... ....
... .. ...
.. ... ....... ..... ...
..... .. .
... . ..... .
. .
...... .. .. .
... . .......... ............. ... . .
.
....................................................................
.
............................................................ .

..................................................

K1 K2 K3 K4 K5

La subgrafica inducida por un subconjunto V 0 del conjunto de vertices es (V 0 , A0 ),


donde si u, v V 0 y uv A entonces uv A0 . Una subgrafica generadora es aquella que
tiene todos los vertices de G.

En demostraciones por induccion, muchas veces conviene considerar las graficas obtenidas
de la original al quitar un vertice o una arista. Si v V, la subgrafica de G que tiene por
conjunto de vertices a V \ {v} y por conjunto de aristas a A \ {a A : v a} se denota
por G v. Si a A, la subgrafica (V, A \ {a}) de G se denota por G a. Tambien, si u y v
son vertices de G no adyacentes, la grafica G + uv tiene por conjunto de vertices a V y por
conjunto de aristas a A{uv}. Analogamente podemos considerar G S para S subconjunto
del conjunto de aristas o vertices.

10.8 Proposicion. Si G es una grafica entonces ella o su complemento es conexa.


Demostracion. Primera forma. Supongamos que G no es conexa. Probaremos que cuales-
quiera dos vertices estan conectados en G. Tomemos u, v V distintos. Si u y v estan en
diferente componente conexa de G, entonces hay una arista entre ellos en G. Si u y v estan en
la misma componente conexa de G, sea w un vertice en otra componente (que existe porque
estamos suponiendo que G no es conexa. Entonces, en G hay una arista de u a w y tambien
de v a w y, por tanto, (u, w, v) es una uv-trayectoria en G.

52
Segunda forma. Procedemos por induccion sobre el orden n de G. Para n = 1 el resultado
es obvio. Supongamos entonces que n 2. En Kn pintemos las aristas de G de verde y las
aristas de G de rojo. Si un vertice u tiene todas sus aristas de un mismo color, entonces u
esta unido con todos los demas vertices en la grafica de ese color, as que el resultado es
cierto. Supongamos entonces que u tiene aristas de cada uno de los dos colores. Por hipotesis
de induccion, en Kn u, alguna de las dos subgraficas resultantes G u o G u es conexa.
Pero a u le llegan aristas de ambos colores as que esta unido con las dos graficas y, en
consecuencia, G o G es conexa.

10.9 Ejercicio. Encontrar una grafica G en la que ambas G y G sean conexas.

10.10 Ejercicio. Probar que solo hay dos graficas 4-regulares de orden 7. Cuantas
graficas 6-regulares hay de orden 9?

Una grafica conexa sin ciclos se llama arbol. Un bosque es una grafica cuyas compo-
nentes conexas son arboles. Un vertice de grado 1 es una hoja.

10.11 Lema. Un arbol tiene al menos dos hojas.


Demostracion. Una trayectoria maximal (en el sentido de que no esta contenida propia-
mente en ninguna otra trayectoria) tiene sus extremos de grado 1.

10.12 Proposicion. Una grafica conexa con n vertices es arbol si, y solo si, tiene n 1
aristas.
Demostracion. () Induccion sobre n. Si n = 1, el numero de aristas es 0 = n 1.
Supongamos n 1. Sea u vertice de grado 1. Entonces G u es arbol con n 1 vertices,
por tanto, por hipotesis de induccion, tiene n 2 aristas, as que el arbol que tenamos tiene
n 1 aristas.

() Supongamos que la grafica tiene k 1 ciclos y quitemos una arista de un ciclo (la grafica
permanece conexa); repitamos esto hasta eliminar todos los ciclos; obtenemos un arbol con
n vertices y n 1 k < n 1 aristas, lo cual es una contradiccion.

10.13 Ejercicio. Probar que las siguientes tres propiedades son equivalentes para una
grafica conexa G:
(a) G es arbol.
(b) Si a G se le quita una arista cualquiera, la grafica resultante es disconexa.
(c) Entre cualesquiera dos vertices hay exactamente una trayectoria.

10.14 Ejercicio. () En cada uno de los casos siguientes explicar por que no se puede

53
construir una grafica G con las condiciones indicadas.
(a) Que los grados de los vertices sean (9, 6, 6, 4, 3, 3, 1).
(b) Que los grados de los vertices sean (7, 4, 4, 4, 3, 3, 2, 1, 1).
(c) Que sea arbol y los grados de los vertices esten dados por la sucesion (6, 5, 4, 2, 2, 1, 1, 1).

10.15 Ejercicio. () Cuantos arboles no isomorfos hay con 6 vertices.

10.16 Ejercicio. () Es posible encontrar dos arboles no isomorfos en que la sucesion


de grados sea la misma?

10.17 Ejercicio. Probar que toda grafica conexa tiene arbol generador.

Decimos que G es bipartita si V se puede partir en dos conjuntos (ajenos) U y W, no


vacos, de manera que todo elemento de A tiene un extremo en U y otro en W. Para m y n na-
turales la grafica completa bipartita, Km,n , consta de m+n vertices u1 , . . . , um , w1 , . . . , wn
y las mn aristas ui wj con i [m] y j [n].

10.18 Proposicion. Una grafica es bipartita si, y solo si, no tiene ciclos de longitud
impar.
Demostracion. () Coloreemos los vertices de uno de los conjuntos de rojo y el del otro
de azul. En cualquier ciclo los vertices estan coloreados alternadamente, as que no puede
haber ciclos de longitud impar.

() Sin perdida de generalidad G es conexa. Supongamos que no tiene ningun ciclo de


longitud impar. Tomemos un vertice cualquiera v y pintemoslo de azul. Ahora pintemos
todos los vertices unidos a v de rojo; posteriormente pintemos todos los vertices unidos a
estos de azul, etc. El que no haya ciclos impares nos dice que si un vertice ya esta pintado
de un color no trataremos de pintarlo del color contrario. La coloracion nos da la particion
buscada.

Sea G conexa. Si u, v V, entonces su distancia es:


d(u, v) = min{l(T ) : T es uvtrayectoria}.
(Para graficas no conexas podra definirse d(u, v) = |V| cuando u y v pertenecen a diferente
componente conexa.)

54
10.1. Paseos eulerianos y ciclos hamiltonianos

Un paseo euleriano en una multigrafica es un paseo que pasa por todas las aristas de
G. Una multigrafica es euleriana si tiene paseo euleriano cerrado.

10.19 Teorema. Las siguientes son propiedades equivalentes para una multigrafica
conexa G.
(a) G es euleriana.
(b) Todo vertice de G tiene grado par.
Demostracion. (a) (b) Es claro, porque cada vez que se pasa por un vertice se usan
dos aristas.
(b) (a) Probemos esto por induccion sobre el numero de aristas. La demostracion de
la base de induccion (0 aristas) es clara. Sea G una grafica con a 1 aristas y 0 vertices
de orden impar. Tenemos la hipotesis de induccion siguiente: En una grafica conexa con 0
vertices de orden impar y menos de a aristas se puede construir un paseo euleriano iniciando
en cualquier vertice y y cualquier arista incidente a el. Utilizando esta hipotesis construyamos
un paseo en G iniciando en cualquier vertice v. Para esto tomemos cualquier arista que tenga
un extremo en v y despues una arista que inicie donde esa arista termino; continuemos as
sucesivamente mientras podamos ir escogiendo aristas sin repetir. En el momento en que
nuestro paseo no pueda continuar sera porque llegamos a un vertice en el que todas las
aristas ya se han usado; como todos los vertices tienen orden par, eso querra decir que el
recorrido se cerro, es decir, que llegamos al vertice v (donde habamos empezado). En este
momento es posible que todava no se hayan usado todas las aristas; sin embargo, cada una
de las partes conexas de la grafica que no hayan sido recorridas es una grafica en la que
todos los vertices son de orden par y que tiene menos aristas que nuestra grafica. Entonces,
consideremos un vertice w del recorrido ya construido en el que haya alguna arista todava
no usada y sea C la parte conexa de la grafica que contiene a w y que todava no ha sido
recorrida. Por la hipotesis de induccion, en C hay un recorrido euleriano que inicia en w.
Entonces a nuestro recorrido le intercalamos esa parte (que obviamente debe terminar en
el mismo w). Podemos hacer lo mismo en todos los vertices donde haya aristas no usadas
todava, obteniendo as el recorrido euleriano.

55
10.20 Corolario. Una multigrafica G tiene paseo euleriano abierto si, y solo si, tiene
exactamente dos vertices de grado impar.
Demostracion. Agregamos arista entre esos vertices y, usando 10.19, construimos un paseo
empezando en una de esas aristas; despues retiramos del paseo la arista agregada.

10.21 Observacion. De la demostracion del teorema es claro que el paseo puede


empezar en cualquier arista si todos los vertices tienen grado par y en, el otro caso, en
cualquier arista de cualquier vertice de grado impar.

10.22 Ejercicio. () El problema de los puentes de Koninsberg describe 4 trozos


de tierra A, B, C y D, separados por un ro pero unidos por algunos puentes, como se
muestra en el esquema. Es posible pasar de pedazos de tierra a otros usando cada puente
exactamente una vez?

10.23 Ejercicio. () (a) Sea G una grafica regular en la que los vertices tienen grado
4. Probar que es posible colorear las aristas de G de manera que a cada vertice lleguen dos
aristas azules y dos rojas.
(b) Probar que si en G todos los vertices tienen grado 6, entonces no necesariamente es
cierto que se puedan pintar de manera que a cada vertice lleguen tres aristas azules y tres
rojas.

As como hablamos de caminos eulerianos, los cuales usan todas las aristas, podemos
considerar caminos hamiltonianos; son aquellos que usan todos los vertices. As habla-

56
mos de ciclo hamiltoniano o de trayectoria hamiltoniana. Una grafica G es grafica
hamiltoniana si tiene ciclo hamiltoniano.

10.24 Ejemplo. Para n 3, Kn y Cn son hamiltonianas.

No existe respuesta completa sobre la existencia de ciclos hamiltonianos en una grafica.


El siguiente es un criterio que podemos usar para determinar que ciertas graficas no son
hamiltonianas.

Dada una grafica G denotamos por k(G) al numero de componentes conexas de G.

10.25 Proposicion. Si G es hamiltoniana, entonces para todo S V no vaco se tiene


que k(G S) |S|
Demostracion. Si C es un ciclo hamiltoniano en G entonces k(G S) k(C S) |S|.

El recproco del criterio 10.25 es falso; en general se usa en forma negativa, es decir,
cuando en una grafica es posible encontrar un subconjunto S del conjunto de vertices tal
que al quitarlo el numero de componentes es mas grande eque el numero de elementos de S,
entonces se puede asegurar que la grafica no es hamiltoniana. Ilustramos esto en es siguiente
ejemplo.
10.26 Ejemplo. La grafica G cuyos vertices son los cuadros de un tablero de ajedrez
de 4 4 con una arista entre dos vertices si, y solo si, los cuadros correspondientes se pueden
alcanzar con un salto de caballo no es hamiltoniana.
Solucion. En el dibujo se ilustra un conjunto S con cuatro elementos tal que al quitar S
de la grafica, el numero de componentes es 6.

10.27 Teorema. (Teorema de Ore.) Si G es una grafica con n 3 vertices en la que


para cada par de vertices u y w no adyacentes se tiene que (u) + (w) n, entonces G es
hamiltoniana.
Demostracion. Supongamos que G no tiene ciclo hamiltoniano y agreguemos todas las
aristas que podamos sin que se forme un ciclo hamiltoniano. En la nueva grafica la condicion
de que la suma de grados de vertices no adyacentes es mayor o igual que n se satisface as
que podemos suponer que la misma G es tal que al agregarle cualquier arista se forma un
ciclo hamiltoniano. Sean u y w dos vertices entre los cuales no hay arista (existen porque en
Kn hay ciclo hamiltoniano). Como al agregar la arista uw se formara un ciclo hamiltoniano,
eso quiere decir que hay una trayectoria hamiltoniana (u = v1 , . . . , vn = w). Ahora, como

57
(u) + (w) n, existe un ndice i tal que uvi+1 y vi w son ambas aristas puesto que, en
caso contrario, consideremos la funcion f : N (w) V \ N (u) dada por f (vi ) = vi+1 que es
inyectiva, y tenemos que N (u) Im(f ) = y que u / N (u) Im(f ), de donde
(u) + (w) = |N (u)| + |N (w)| = |N (u)| + |Im(f )| n 1,
lo cual es una contradiccion. Tomemos entonces un ndice i tal que uvi+1 A y vi w A e
invertamos la porcion de la trayectoria de vi+1 a w para obtener el ciclo:
(u = v1 , v2 , . . . , vi , w = vn , vn1 , . . . , vi+1 , v1 = u),
de donde tenemos una contradiccion.

10.28 Ejercicio. A una comida van a asistir n 3 embajadores. Cada uno de ellos
es amigo de al menos n2 embajadores. Probar que se pueden sentar en una mesa redonda de
manera que cada embajador este al lado de dos de sus amigos.

10.29 Ejercicio. () Determinar si las siguientes graficas (graficas de los solidos


platonicos) son hamiltonianas o no.

10.30 Ejercicio. () Sea Gm,n la grafica cuyos vertices son los cuadros de un tablero
rectangular de ajedrez de m n con una arista entre dos vertices si, y solo si, los cuadros
correspondientes se pueden alcanzar con un salto de caballo. Usar 10.25 para probar que si
m es impar, entonces Gm,m no es hamiltoniana.

58
10.31 Ejercicio. () Decir cual es el error en la siguiente demostracion del problema
de los embajadores 8.6: Procedemos por induccion sobre n. Para n = 3 es cierto pues n2 = 32 ,
as que cada embajador debe tener al menos dos amigos y la grafica de amistades es un
triangulo. Consideremos ahora el caso de n embajadores con n 4; usando la hipotesis de
induccion sentemos a n 1 de los embajadores en mesa redonda y sea E el embajador que
falta por sentarse. Como el tiene al menos n2 amigos hay dos embajadores sentados juntos
que son amigos de E y entonces el se puede sentar entre ellos.

10.32 Ejercicio. () Hay un tesoro en cada cubo de 1 1 1 de los 343 que forman un
cubo de 7 7 7. Un duende se encuentra en el cubo central; en cada momento puede pasar
de un cubo a cualquier otro que comparta un cuadrado con el cubo donde esta. Resulta que
si regresa a un cubo por el que ya paso, entonces un monstruo le quita todos los tesoros que
ha obtenido hasta el momento. Las salidas estan en las 8 esquinas del cubo. Es posible que
salga del cubo con todos los 343 tesoros?

10.2. Graficas planas

Decimos que G es una (multi)grafica plana si esta dibujada en el plano (o, equivalen-
temente, en la superficie de una esfera) y sus aristas no se intersectan (salvo en los vertices
que las forman).

Una (multi/seudo) grafica G es aplanable si es isomorfa a una (multi/seudo) grafica


plana.

10.33 Ejemplo. K4 es aplanable.

En una grafica plana cualquier region delimitada por aristas es una cara, incluso la region
exterior (no acotada).

En dos graficas planas isomorfas es posible que las caras no tengan el mismo numero de
lados, como lo muestra la siguiente figura.

59
10.34 Ejemplo. Son aplanables Cn , los arboles y las graficas de los solidos platonicos.

10.35 Ejercicio. () Probar que K2,n es aplanable.

En lo que sigue, denotamos por V , A y C a los numeros de vertices, aristas y caras,


respectivamente, de una (multi/seudo) grafica plana G.

10.36 Teorema. (Formula de Euler.) Si G es conexa, entonces

V A + C = 2.
Demostracion. Induccion sobre A. Por ser la grafica conexa, el menor numero de aristas
que puede tener es A = V 1, cuando G es un arbol. En este caso solo hay una cara, de
donde se satisface trivialmente la formula. Ahora tomemos una grafica G con A aristas, donde
A V . Entonces G tiene ciclos; sea a una arista perteneciente a un ciclo y consideremos la
grafica G 0 obtenida de quitar a a G. Observemos que al quitar la arista a, alguna cara se une
con otra (todo esto, incluso si a es lazo o arista multiple), de manera que

V 0 = V,
A0 = A 1,
C 0 = C 1,

donde V 0 , A0 y C 0 son, respectivamente, los numeros de vertices, de aristas y de caras de


G 0 , y as, usando la hipotesis de induccion, es claro que la formula se satisface tambien para
G.

10.37 Corolario. El numero de caras de una (multi/seudo) grafica aplanable conexa


no depende del dibujo en el plano.

10.38 Ejercicio. Usar la formula de Euler para determinar el numero de regiones en


que n rectas en posicion general dividen al plano.

10.39 Proposicion. Si G tiene al menos 3 vertices y es grafica aplanable maxima (es


decir, al agregar una arista ya no es aplanable) entonces todas sus caras son triangulos.
Demostracion. Dibujemos G de manera plana en R2 , y supongamos que alguna cara tiene
mas de 3 lados; entonces se puede agregar una arista entre dos vertices no consecutivos de
esa cara y la grafica sigue siendo plana, lo cual es una contradiccion.

60
10.40 Proposicion. Si G es grafica aplanable con V 3 vertices y A aristas entonces

A 3V 6.
Demostracion. Construyamos G 0 a partir de G agregando aristas de manera que G 0 sea
plana maxima (y conexa). Si V 0 , A0 y C 0 son los numeros de vertices, aristas y caras de G 0 ,
0
respectivamente; entonces 3C2 = A0 (pues cada cara tiene tres aristas y cada arista pertenece
a dos caras), de donde C 0 = 32 A0 ; sustituyendo en la formula de Euler tenemos
2 1
2 = V 0 A0 + A0 = V 0 A0 .
3 3
Pero V = V 0 y A A0 , de donde
1 1
V A V 0 A0 = 2,
3 3
y de aqu tenemos el resultado buscado.

10.41 Corolario. K5 no es aplanable.


Demostracion. Si fuera aplanable cumplira que A 3V 6, pero 3V 6 = 3 5 6 =
9 < 10 = A.

10.42 Observacion. Si H es subgrafica de una grafica aplanable, entonces H es apla-


nable.

10.43 Corolario. Kn no es aplanable para n 5.

10.44 Ejercicio. () Probar que K3,3 no es aplanable usando el teorema de Euler.

10.45 Corolario. Si G es aplanable entonces tiene al menos un vertice de grado menor


o igual que 5. P
Demostracion. Si todos tuvieran grado mayor o igual que 6, entonces 2A = v (v) 6V,
de donde A 3V > 3V 6.

10.46 Nota. Como acabamos de ver, K5 y K3,3 no son aplanables ni lo es ninguna


grafica que contenga alguna de ellas como subgrafica. Una especie de recproco de este
resultado es cierto. Es el teorema de Kuratowsky: Si G es aplanable, entonces G no contiene
a K5 ni a K3,3 . Sin embargo hay que interpretar lo de que no contiene de cierta manera;
por ejemplo, es claro que si a K5 le agregamos un vertice enmedio de una arista, la nueva
grafica tampoco sera aplanable a pesar de no contener a K5 ni a K3,3 . Tambien, por ejemplo,
es claro que la grafica de petersen dibujada aqu abajo no es aplanable pues si contrajeramos
a puntos las aristas que unen a los dos pentagonos obtendramos K5 . No haremos preciso
aqu el sentido de no contiene ni probaremos el teorema.

61
La grafica dual de una (multi/seudo) grafica plana conexa G es (multi/seudo) grafica

G construida a traves de G poniendo un vertice por cada cara de G y tal que dos vertices de
G estan unidos mediante una arista si, y solo si, las caras correspondientes en G comparten
arista.

10.47 Observacion. (a) Como vimos en la figura, aun cuando G sea simple, G no
necesariamente lo es.
(b) G tambien es aplanable porque por cada cara de G solo hay un vertice y por cada
arista de G hay exactamente una arista de G , as que, dentro de cada cara, las aristas se
pueden dibujar de manera que no haya interseccion (salvo en el vertice que representa la
cara).

62
(c) La grafica dual de G es G.
(d) La grafica dual depende del encaje en el plano. Por ejemplo, las dos graficas G1 y G2
dibujadas abajo son isomorfas, pero observemos que las dimensiones de las caras de G1 son
(5, 5, 4, 3), mientras que las de G2 son (6, 4, 3, 3), lo cual hace que en G1 no haya vertice de
grado 6, mientras que en G2 s lo hay.

Como primera aplicacion de lo anterior probaremos que los unicos poliedros regulares son
los platonicos.
Se puede abstraer la definicion de poliedro de diversas maneras. Nos interesa aqu anali-
zarlos como graficas y considerar algunas propiedades basicas, las cuales mencionamos aqu:
Son graficas conexas, planas (mas bien las consideramos dentro de la esfera), cada cara esta
definida por un ciclo de longitud al menos 3 y cada vertice tiene grado al menos 3. A una
grafica que cumple estas propiedades le llamamos poliedrica.

10.48 Observacion. La grafica dual de una grafica poliedrica tambien es poliedrica.

10.49 Teorema. Los unicos poliedros regulares son los platonicos.


Demostracion. Supongamos que G es una grafica poliedrica regular en la que el grado de
cada vertice es g y el numero de aristas de cada cara es l. Sea V el numero de vertices, A
el numero de aristas y C el numero de caras de G. Por la definicion de grafica poliedrica
tenemos que
g, l {3, 4, 5}.
Ademas,
Vg Cl
A= = .
2 2
Consideremos los distintos casos para g:
Si g = 3, entonces 3V2 = A = Cl2
, de donde V = Cl
3
y en la formula de Euler Cl
3
Cl
2
+C =
2 y, por tanto, C(6 l) = 12. Para l = 3 se tiene C = 4, A = 6 y V = 4; este es el caso del
tetraedro. Para l = 4 se tiene C = 6, A = 12 y V = 8; este es el caso del cubo. Para l = 5
se tiene C = 12, A = 30 y V = 20; este es el caso del dodecaedro.
Si g = 4, entonces 4V2 = A = Cl2
, de donde V = Cl
4
, y en la formula de Euler Cl
4
Cl
2
+C =
2, as que C(4 l) = 8. Aqu la unica posibilidad es l = 3, y entonces C = 8, A = 12 y
V = 6; este es el caso del octaedro.
Si g = 5, entonces 5V2 = A = Cl2
, de donde V = Cl
5
, y en la formula de Euler Cl
5
Cl
2
+C =
2, y tenemos C(10 3l) = 20. Aqu la unica posibilidad es l = 3, y entonces C = 20, A = 30
y V = 12; este es el caso del icosaedro.

63
Ahora consideraremos otra aplicacion importante: la coloracion de mapas. El problema
consiste en lo siguiente: Dado un mapa (de pases), cuantos colores bastan para colorear
cada pas de manera que si dos pases comparten una frontera entonces su color es distinto?
El problema traducido a graficas es el siguiente: Dada una grafica plana en la que no hay
vertices de grado 1, cuantos colores bastan para pintar sus caras de manera que caras
adyacentes tengan distinto color? Se ha demostrado, mediante el uso de la computadora,
que 4 colores bastan. Nosotros lo probaremos aqu para 5 colores. Consideraremos el caso
dual (coloracion de vertices es decir, funcion del conjunto de vertices en un conjunto cuyos
elementos se llaman colores), recordando que los lazos en la grafica dual de una grafica se
producen cuando un vertice de la grafica original tiene grado 1. Decimos que una coloracion
de vertices es buena o propia si vertices adyacentes tienen distinto color.

10.50 Proposicion. Sea G una grafica plana sin lazos. Entonces existe una buena
coloracion con 5 colores para los vertices de G.
Demostracion. Sin perdida de generalidad podemos suponer que la grafica es conexa.
Tambien es claro que podemos suponer que no hay aristas multiples. Supongamos entonces
que G es una grafica simple conexa. Procedemos por induccion sobre V . Si V = 1 no hay
nada que probar. Supongamos que V > 1 y que el resultado es cierto para graficas con menos
de V vertices. Si la grafica no es poliedrica, por ser simple y conexa debe de tener un vertice
de grado menor que 3; si la grafica es poliedrica entonces, tiene un vertice de grado menor
o igual que 5; en cualquier caso, sea v un vertice de grado menor o igual que 5 y usemos la
hipotesis de induccion para colorear los vertices de G v con 5 colores de manera que no
haya vertices adyacentes con el mismo color. Es claro que si alguno de los 5 colores no se uso
en los vertices adyacentes a v, entonces podemos pintar v con uno de ellos, obteniendo as
la buena coloracion buscada para G. Veamos que existe una coloracion para los vertices de
G v en la que en los vertices adyacentes a v no se usan los 5 colores. Supongamos que no
y sean v1 , v2 , . . . , v5 los vertices adyacentes a v, escritos en orden de las manecillas del reloj;
sea i el color usado para vi . Por nuestra suposicion, el color 3 no puede usarse para v1 (pues
v3 lo usa); eso quiere decir que v1 esta unido a algun otro vertice ya pintado con el color 3,
pero analogamente, ese otro vertice no puede cambiarse de color al 1 por estar unido a otro
con el numero 3 y as sucesivamente; la unica posibilidad de que no pueda intercambiarse el
color 1 por el 3 en toda la grafica sin cambiar tambien el color de v3 es que se forme una
trayectoria de v1 a v3 en G v en que los vertices vayan alternando los colores 1 y 3. Esto

64
mismo puede hacerse con los vertices v2 y v4 ; sin embargo esto produce dos trayectorias que
se intersectan; como la grafica es plana, la interseccion es un vertice comun a los caminos,
lo cual es un absurdo.
1 3
...
. .... .... .... .... .... .... .... ....
....
.... ....
....
.... ....
.. 1
. ....
... ....
..
.
... . . .. .. .... ... . .... .... .... .......
.
... .. .. .....
. . .. ...
....
.
...
...
v2... .... .... ...
. ...
4 .
... 2 ...
...
3 ........ .......... .
... 2 ...
..
..
...
...
.. ... .. ...
..
.
...
...
... 3 ..
.....
.
.
....... 4
...
...
...
. .
........ .
............
v 3 ..
... .... ...... ...
......
v1 ....................... ...
...
... ..
..
.......
......... ...
...
..
1 ..................................................................... ...

.
....................................
... .
... .... . 2
... .
..

... v
.. ..........
..
. ..........
.......... 4
.......... .... ....
....
.
...

..
...
.
..
.

.......

...
..
.
v 4
v5 .... 5

10.51 Ejercicio. () Explicar por que no puede continuarse la demostracion de 10.50


para que, una vez cambiado el color de v3 por el color 1, continuar con un argumento similar
para intentar cambiar el color de v5 por el de v3 (o el de v4 por el de v2 ), y entonces concluir
que 4 colores son suficientes.

10.52 Ejercicio. () Encontrar una grafica plana sin lazos cuyos vertices no tengan
una buena coloracion con 3 colores.

65
Referencias y lecturas complementarias
Bona, M., A Walk through Combinatorics, World Scientific, 2002.

Bondy J.A., Murty, U.S.R., Graph Theory with Applications, The Macmillan Press Ltd,
1977.

Engel, A., Problem-Solving Strategies, Springer, 1997.

Lovasz, L., Combinatorial Problems and Exercises, AMS Chelsea Publishing, American
Mathematical Society Providence Rhode Island, 2nd edition, 2007.

Perez Segu M. L., Combinatoria, Cuadernos de Olimpiadas de Matematicas, Instituto


de Matematicas, UNAM, 3a edicion, 2009.

Perez Segu M. L., Combinatoria Avanzada, Cuadernos de Olimpiadas de Matematicas,


Instituto de Matematicas, UNAM, 2010.

Soulami, T.B., Les Olympiades de Mathematiques, Reflexes et Strategies, Ellipses Edi-


tion Marketing S.A., 1999.

Vilenkin, N., De Cuantas Formas? (Combinatoria), Editorial Mir, Moscu, 1972.

66

You might also like